Wikipedia:Reference desk/Humanities
of the Wikipedia reference desk.
Main page: Help searching Wikipedia
How can I get my question answered?
- Select the section of the desk that best fits the general topic of your question (see the navigation column to the right).
- Post your question to only one section, providing a short header that gives the topic of your question.
- Type '~~~~' (that is, four tilde characters) at the end – this signs and dates your contribution so we know who wrote what and when.
- Don't post personal contact information – it will be removed. Any answers will be provided here.
- Please be as specific as possible, and include all relevant context – the usefulness of answers may depend on the context.
- Note:
- We don't answer (and may remove) questions that require medical diagnosis or legal advice.
- We don't answer requests for opinions, predictions or debate.
- We don't do your homework for you, though we'll help you past the stuck point.
- We don't conduct original research or provide a free source of ideas, but we'll help you find information you need.
How do I answer a question?
Main page: Wikipedia:Reference desk/Guidelines
- The best answers address the question directly, and back up facts with wikilinks and links to sources. Do not edit others' comments and do not give any medical or legal advice.
September 23
Corporate tax rate and hiring costs
I've run across a couple blog posts ([1] and [2]) which confirm the recently empirically validated idea that decreasing the effective corporate tax rate tends to suppress hiring because it puts the return from paying taxes on profits above the expected return from hiring and further production using pre-tax revenue (which also involves a feedback threshold effect in the macroeconomic situation when many businesses are facing the same changes in the effective tax rate -- if they all aren't hiring, the risk of investing in further production increases because consumer spending is suppressed.) As we've seen, this is accompanied by a spike in profits and bank deposits. I've come across some very oblique references to this effect in old theory papers, and in one recent labor economics sourcebook, but I'm looking for more mainstream sources on it. Does anyone have any suggestions for where this particular situation might have been studied in peer reviewed papers? —Cupco 00:54, 23 September 2012 (UTC)
- I don't think you will find much mainstream support for those ideas. On the other hand, hopefully the additional taxes collected aren't just used to line politician's pockets. If those tax dollars are used to fund education, infrastructure, etc., this can create a more attractive place to open or expand a business, if they have an educated workforce, good roads, ports, airports, etc. StuRat (talk) 01:03, 23 September 2012 (UTC)
- StuRat, I'm reasonably certain you didn't understand the question — it doesn't claim that there are "additional taxes collected" at all, it is about decreasing the tax rate — and using it as an opportunity to stand on a generic soap box about taxation is very poor form indeed. (It looks like you are trying to start a debate on an entirely different question.) If you don't know the answer — it would take some familiarity with economics to give a good answer — why not do the world a favor and keep it to yourself? --Mr.98 (talk) 01:47, 23 September 2012 (UTC)
- Those blogs argue that decreasing the corporate tax rate suppresses hiring. Therefore, we are comparing a lower corporate tax rate with a higher one, to try to determine which will create more jobs. In that context, we should consider both sides of the equation, how the increased (or decreased) taxation affects hiring, and also how the increased (or decreased) spending affects job growth. It's illogical to consider the impact of taxation without also looking at the impact of the corresponding spending. Indeed, the ideal tax rate for job creation is entirely dependent upon how the government spends those taxes. StuRat (talk) 03:09, 23 September 2012 (UTC)
- Except of course you need to first establish a decreased corporate tax rate leads to a low level of government tax/revenue collection (and therefore less money available for government spending) which wasn't at issue until you started to bring up off topic issues in to the discussion but could easily be disputed since as per the original discussion, the effect of changing the tax rate is complicated. Nil Einne (talk) 04:59, 23 September 2012 (UTC)
- It has been argued that, in the long run, cutting taxes can increase revenue, but not in the short run. (If you cut taxes by 10%, that's not going to increase the size of the economy by 10% the first year.) And, of course, there's a limit to the benefits of always cutting taxes, as obviously a 0% tax rate won't increase tax revenue, no matter what the economy does. StuRat (talk) 06:13, 23 September 2012 (UTC)
- Wasn't that Mr 98's point? -- ♬ Jack of Oz ♬ [your turn] 06:20, 23 September 2012 (UTC)
- It was also partially my point, but StuRat is mistaken anyway. It's not simply a matter of growth which may take some time, but changes to behaviour which can happen very fast, e.g. if a company starts bringing more money in to the US which they used to hide in other places. While it's not usually claimed it will balance out straight away except in extreme cases, it is ultimately incredibly complicated and not something even economists agree on so not something to be just dismissed as unimportant. In any case, most of the spending StuRat described like education etc will take a long time to have an effect. In other words, if you want to consider this, you have do actually consider it properly and not just throw in random extra information. Either that or we can just stick with the original question and assume the OP is aware it's incredibly complicated. Nil Einne (talk) 07:47, 23 September 2012 (UTC)
- While the benefits will last for decades, an immediate benefit in improved employment due to hiring more teachers, construction workers, road crews, etc., will also be felt. StuRat (talk) 22:27, 23 September 2012 (UTC)
- Hence the key word 'most'.... Nil Einne (talk) 07:17, 24 September 2012 (UTC)
- While the benefits will last for decades, an immediate benefit in improved employment due to hiring more teachers, construction workers, road crews, etc., will also be felt. StuRat (talk) 22:27, 23 September 2012 (UTC)
I was trying to ask about scholarly studies of the cost of paying tax on profits compared to the expected returns of hiring with pre-tax revenue in the macroeconomic context of a uniformly decreasing effective business tax rate. I was not trying to ask about the relationship between the tax rate and economic growth, which has been studied and written about extensively, almost entirely without the benefit of information pertaining to the former. I do not think this is very complicated, it's just a matter of finding the right keywords and source citations. —Cupco 03:14, 24 September 2012 (UTC)
- Unfortunately, I believe the arguments in those blogs to be a fringe theory, unlikely to be supported in scholarly research. StuRat (talk) 08:21, 24 September 2012 (UTC)
- On what basis? They are simple math anyone can verify with a calculator and a copy of the Internal Revenue Code. —Cupco 13:06, 24 September 2012 (UTC)
- It's the faulty underlying assumption: "Businesses want to make X dollars, so if we tax them more, they will do more so they can still earn X". The more reasonable assumption is "Businesses want to make as much money as possible". In this case, they hire as many people as needed to maximize profits, regardless of the tax rate. StuRat (talk) 20:19, 25 September 2012 (UTC)
- You are misunderstanding the situation. If the effective tax rate is 30% and the return on investment from hiring and production is 5%, then hiring makes sense because it can be done with pre-tax revenue. If the effective tax tate is 10% and the expected return from hiring and production is -10% because of decreased consumer demand (because businesses aren't hiring...) then it makes more sense (in terms of the "make as much money as possible" goal) to pay the tax and bank the after-tax profits. Even if the expected return from hiring and production is thought to be -10% nominally, if the effective tax rate is 30% then it still makes more since to hire instead of bank, and if all businesses are facing that same decision, then aggregate demand will increase because of the additional consumer spending. —Cupco 22:05, 25 September 2012 (UTC)
Evangelicalism in the US
I've wondered why do evangelicals seem to have so much power in the United States. It seems that public influence can be swayed by evangelicals, and politicians often campaign to get their votes. There's even a fast-food restaurant that closes on Sundays, which is quite unusual in current society. Evangelicals also seem to be much more common (or at least more visible or influential) in the United States than in any other country. Why is this the case? Does it have to do with the fact that, in the early days of the United States, people emigrated there to practice their religion and escape persecution? Or maybe this is just an illusion, since I've watched too much Simpsons? Narutolovehinata5 tccsdnew 02:09, 23 September 2012 (UTC)
- Numbers. Evangelical Christians are a large enough proportion of the population in some parts of the U.S. to be able to be a sizable voting block. In the Southeastern U.S., your question would be like asking "Why do Muslims have so much power in Egypt?" Of course, in many parts of the U.S. (Northeast, Midwest, West Coast, etc.), there are not many Evangelical Christians, and so there aren't as many Evangelical politicians. --Jayron32 02:56, 23 September 2012 (UTC)
- "...(Northeast, Midwest, West Coast, etc.), there are not many Evangelical Christians"? That's just not true. California, for example, has more Evangelical Christians than any other state. However, they are proportionally fewer in the West and Northeast (I wouldn't include the Midwest) and so not as important as a voting block.--William Thweatt TalkContribs 04:38, 23 September 2012 (UTC)
- California has more of everything than any other state. It's 1/6th of the country by population. Evangelicals in California don't make as significant of a voting block as they do in, say, Alabama, because they are not as significant a portion of the Californian population. If I crammed together a bunch of southern states to equal the same population as California, there'd be more Evangelicals in that southern Mega-state. --Jayron32 04:55, 23 September 2012 (UTC)
- Agreed. One thing that must be understood about US politics is that evangelicals have large negatives with a significant portion of US voters. You will note that the Republican Party is trying very hard to hide their evangelical block from the public, as they know this is a sure way to lose a national election. StuRat (talk) 03:14, 23 September 2012 (UTC)
- Sources? I'd say it's far more reasonable to state that parties are trying to hide their more radical subgroups, but "evangelical" is not a one-to-one correspondence with "radical". — Lomn 13:20, 23 September 2012 (UTC)
- True. Just as there was no mention of the "Tea Party" at the Republican Convention, I wouldn't expect much mention of the "99 Percenters" at the Democratic Convention, either. StuRat (talk) 18:35, 23 September 2012 (UTC)
- (Additionally, I'd say that things like both parties including "Jerusalem is the rightful capital of Israel", or however it's phrased, in their national platforms is calculated to appeal to a large number of evangelical Christians -- even the radical ones -- as well as Jews) — Lomn 13:22, 23 September 2012 (UTC)
- That specific phrase is meant to reassure both Evangelicals and Jews while being relatively unimportant to less conservative Protestants, Roman Catholics, and most others - with the obvious exception of Muslims, who don't constitute a large enough voting bloc for the politicians to take into consideration. --NellieBly (talk) 21:47, 23 September 2012 (UTC)
- (Additionally, I'd say that things like both parties including "Jerusalem is the rightful capital of Israel", or however it's phrased, in their national platforms is calculated to appeal to a large number of evangelical Christians -- even the radical ones -- as well as Jews) — Lomn 13:22, 23 September 2012 (UTC)
Democratic governments act in the interest of their citizens. Politicians in democracies are elected by their citizens. So if a certain district has a large number of Evangelicals, a politician is naturally going to try to recruit them, because he/she will have to represent them in the government. The same would go for a district with any religion, race, ethnicity, etc. Evangelicals are a large part of American society, and as such, have greater influence. That's how a democracy functions. The part about closing on Sunday isn't part of the government. That's a personal decision by the restaurant owner, and can happen in any country even among a minority (the issue of #s doesn't apply here). --Activism1234 03:29, 23 September 2012 (UTC)
- To put this quantitatively, 26% of Americans identify as evangelicals according to our article. Evangelicals also have disproportionate power because they put heavy emphasis on a president's piety. A president can hope to attract the votes of a vast majority of evangelicals by being more religious than the other guy, but can't hope to attract a similar percentage of votes from the non-religious (15% of the population) by being less religious. --140.180.242.9 (talk) 03:50, 23 September 2012 (UTC)
- Yes, but if he espouses the issues of the evangelicals, like saying he wants to ban abortion and arrest all illegal immigrants and their children, that will lose him the national election. StuRat (talk) 03:54, 23 September 2012 (UTC)
- AFAIK, arresting all illegal immigrants and their children is neither covered in the Bible nor a position particularly attributed to Evangelicalism.--William Thweatt TalkContribs 04:38, 23 September 2012 (UTC)
- The Bible doesn't explicitly condemn abortion either and actually values prenatal life as less than postnatal life since I think there was a line there about someone being forced to pay a fine for forcing a woman to miscarry rather than being charged with murder. Also, Reagan and both Bushes supported repealing Roe v. Wade (which would have caused some U.S. states to ban abortion) and they won the U.S. Presidency, sometimes by huge electoral and popular vote margins. Futurist110 (talk) 05:42, 23 September 2012 (UTC)
- Yes, but none of them campaigned on that issue: "A Bush Presidency will mean the end to abortion forever !". That would have made them lose. Keeping their evangelical leanings hidden is the way to go, which is what Romney is doing. Also, back in Reagan's time, the backlash against evangelicals hadn't fully developed yet. As for the views of evangelicals being at odds with the Bible, I couldn't agree more. StuRat (talk) 06:06, 23 September 2012 (UTC)
The concept of separation of church and state in America was a result of reflection on Puritan society. Consider Thomas Hooker, Anne Hutchinson, Roger Williams, etc, who rejected Puritan lifestyle and established new colonies that promoted religious tolerance. The Salem Witch Trials really did it too. --Activism1234 04:51, 23 September 2012 (UTC)
- Actually, the Baptists (evangelical Christians) were a major force for seperation of Church and State in America, Thomas Jefferson corresponded with several Baptist congregations over the issue, with one such letter from Jefferson (Letter to Danbury Baptists )being the original source of the famous phrase "Wall of Separation". See Baptists in the history of separation of church and state. Evangelical christians thus have a long history of supporting seperation of Church and State. --Jayron32 05:21, 23 September 2012 (UTC)
- There appears to be some confusion. I was discussing Puritan society - not Baptists or Evangelicals. --Activism1234 06:26, 23 September 2012 (UTC)
- By the 18th entury, the Puritan movement (originally ultra-Protestant Anglicans who disagreed with keeping Catholic traditions within the church) had formed Baptist and Quaker congregations. So by 1777, Puritans were Baptists. See our article Puritan. 82.153.115.236 (talk) 13:22, 23 September 2012 (UTC)
- There appears to be some confusion. I was discussing Puritan society - not Baptists or Evangelicals. --Activism1234 06:26, 23 September 2012 (UTC)
- The problem I'm having with this question is that the OP has confused "evangelical Christians" with "conservative Christians". It's not an accurate assumption - many, *many* conservative Christians are Catholic (and Orthodox), while many, *many* evangelical Christians are not politically conservative. --NellieBly (talk) 21:47, 23 September 2012 (UTC)
Who made this curse? (shown on PBS about 10-15 years ago)
A man was being pressed to death and at one point his tongue stuck out. Someone took his cane and pushed it back in. Then he put curse on the estate until every stone he scartered over the land is returned. Maybe he said how many. And apparently stuff after that (like the Hpe diamond) that the modern owners keep the one(s) that've been found. And there's lake or pond on the property and they believe that they might never (all) be found. 96.246.70.87 (talk) 03:17, 23 September 2012 (UTC)
- That'd be Giles Corey. --Jayron32 03:21, 23 September 2012 (UTC)
- Are you sure? I could've sworn that (maybe distinctive looking) stones were the curse. 96.246.70.87 (talk) 06:52, 23 September 2012 (UTC)
- The part about the curse is bullshit. The rest of the stort, regarding being pressed to death and the cane being used to put the tongue back in his mouth, is history. Lots of the stuff related to the Salem Witch Trials has been embelished over the years, sometimes with some really fantastical stuff. The actual historical figure cast no curses. The only words he spoke during the ordeal were "more weight". --Jayron32 17:15, 23 September 2012 (UTC)
- Ugh. That's carrying "freedom of the press" a little too far. ←Baseball Bugs What's up, Doc? carrots→ 03:41, 24 September 2012 (UTC)
- The part about the curse is bullshit. The rest of the stort, regarding being pressed to death and the cane being used to put the tongue back in his mouth, is history. Lots of the stuff related to the Salem Witch Trials has been embelished over the years, sometimes with some really fantastical stuff. The actual historical figure cast no curses. The only words he spoke during the ordeal were "more weight". --Jayron32 17:15, 23 September 2012 (UTC)
- Are you sure? I could've sworn that (maybe distinctive looking) stones were the curse. 96.246.70.87 (talk) 06:52, 23 September 2012 (UTC)
Names with ordinal numbers
There is an apparently purely American (other than for royalty and pontiffs) habit of using ordinal numbers with names (e.g. Davis Love III, George Hamilton IV). My central question is "What's up with that?", but it might be useful to break it down into How did it arise? Is it practised anywhere else? and Is it considered a bit pompous even there? I'm genuinely interested, so no Yank-bashing please. HenryFlower 05:56, 23 September 2012 (UTC)
- It denotes the third bearer of that name in that family (in the case of Davis Love) or the 4th in the case of George Hamilton. European monarchs and popes follow the convention. There is, however, an interesting exception: the House of Reuss, whose males are all called Henry and numbered sequentially until you get to 100, when the sequence starts again. --TammyMoet (talk) 08:26, 23 September 2012 (UTC)
- Yes, I know what it means, and I already mentioned that monarchs and popes follow it. That doesn't answer my questions. HenryFlower 09:26, 23 September 2012 (UTC)
- Is there a term for this practice? The closest I can find to relevant articles are Name at birth and wiktionary:name after. 81.98.43.107 (talk) 11:53, 23 September 2012 (UTC)
- As opposed to "Davis Love 3" or "Davis Love the Third"? This is certainly something that has evolved. Shakespeare originally called the monarch in the play we call Henry VIII (play) by the term "Henry the eight." Not "eighth" and not "VIII". ←Baseball Bugs What's up, Doc? carrots→ 13:21, 23 September 2012 (UTC)
- Googling this subject suggests that it's adopted from the tradition of using Roman Numerals with royal lineage. Also, consider this: The typical name consists of strictly letters, not numbers (with the exception of Tom Lehrer's friend Hen3ry). Roman numberals are letters too. So it works. ←Baseball Bugs What's up, Doc? carrots→ 13:36, 23 September 2012 (UTC)
- (or Jennifer 8. Lee 69.228.171.70 (talk) 16:19, 23 September 2012 (UTC))
- OK, so what does the "8." stand for? Although my guess is that it stands for the same thing as the "S." in "Harry S. Truman", or the "H." in "Jesus H..." - well, you get the idea. ←Baseball Bugs What's up, Doc? carrots→ 04:21, 24 September 2012 (UTC)
- (or Jennifer 8. Lee 69.228.171.70 (talk) 16:19, 23 September 2012 (UTC))
- No--the format of the ordinal is not what interests me; its use is (serves me right for trying to be concise). This manner of naming would never* be used in the UK, because a) children seem less often to be named after their parents, and b) if they are, they would be referred to by some sort of nickname rather than the full name. What interests me is the sociology of the American practice.
- To complicate things further, is the practice ever followed with women? (Now that married names are going out of fashion).
- With the exception of a family of eccentric publishers mentioned here a few years ago. HenryFlower 15:28, 23 September 2012 (UTC)
- The practice doesn't exist in the UK because we have our own naming conventions. --TammyMoet (talk) 19:34, 23 September 2012 (UTC)
- It does occasionally occur in Norway. The grandfather would be Sr. (senior), his son would be Jr. (junior), and the grandson when reaching adulthood and some notability, might add a 'III' to his name to avoid confusion. I've seen this in a family where there are three doctors, grandfather, son and grandson all having the same given name and surname. I checked the grandson in pubmed. In early publications, the grandson is listed with a '3rd' behind his last initial. In later publications, he has dropped the '3rd'. His father is still 'Jr.' in pubmed, at a time when 'Sr.' surely must have passed away. --NorwegianBlue talk 20:48, 23 September 2012 (UTC)
- I may be misremembering, but the only Australian case I'm aware of is William Wentworth (Australian politician) (1907-2003), who was generally known as Bill or Billy or plain William, but in historical contexts it was sometimes "William Wentworth IV", to distinguish him from his great-grandfather the explorer William Wentworth (1790-1872). He had a son of the same name (II), and from a different son he had a grandson of the same name (III), neither of whom was notable. William III was the father of William IV. Their middle names couldn't be used to tell them apart because all 4 of them had Charles. -- ♬ Jack of Oz ♬ [your turn] 21:31, 23 September 2012 (UTC)
- Our article onomastics (meaning the study of proper names, anthroponomastics being the study of humans' names) mentions an American Name Society, who publish a journal on onomastics. That might be a good place to look, or you could try contacting the society. 81.98.43.107 (talk) 23:47, 23 September 2012 (UTC)
- As far as the pompousness of it... For the average Joe, it would be pretty pompous to go around insisting that you be referred to by the ordinal. It's generally just used in things that are legal or if fathers and sons are in the same business. I've known a few "thirds" and I've never heard them introduced to anyone with the "third" spoken. If it comes up in conversation, it comes up. But it's rarely pointed out during day to day use. Dismas|(talk) 00:33, 24 September 2012 (UTC)
- Naming conventions in America have varied over the centuries. In early America, if three unrelated men named "John Smith" lived in a small town, they were called "John Smith Sr.," "John Smith Jr.," and "John Smith III" in order of their ages. When "John Smith Sr." died, the "Jr." moved up to "Sr." etc. I don't know when the custom referred to by the OP came into vogue, but I've always assumed it came from Britain. How sure are we that it was never the custom in the UK to name men in a lineage "John Smith Sr.," "John Smith Jr" .(likely nicknamed 'Johnny,'). "John Smith, III" (likely Trey for short) etc? Maybe the Brits did it in the 1600's but do not now because it seems too pretentious. It would be surprising if people in the US started doing it a couple of hundred years ago with no prior history of such a naming custom. Edison (talk) 01:39, 24 September 2012 (UTC)
- In Britain there are peers, who use titles rather than their given/surnames, and most titles come with numbers. The 19th Viscount Gibbetworthy is quickly differentiated from any of his predecessors. Same for ruling monarchs. But people of the lower orders would have been seen to be somewhat pretentious, and getting a bit beyond their station, if they adopted such a practice. In the US, where there is officially no institutionalised class system, I suspect it's only those who've become public figures, or leaders in a certain field, or have inherited wealth and fame and are expected to continue the dynasty, or see themselves as apart from the common herd, who would do this. Would Thurston Howell, III have had the ordinal if he were a nondescript nonentity such as a file clerk on average wages? Hardly. (No offence to nondescript nonentities, or file clerks on average wages.) -- ♬ Jack of Oz ♬ [your turn] 02:10, 24 September 2012 (UTC)
- If the Howells were a poor farming family who named, say, the oldest son after the father in each generation, then Thurston Howell III might be a man of no property or influence, and would not be a subject of ridicule in the US. I have seen such things many times in genealogy research. It is a custom brought from England, like "Herbert Pelham III," born 1600 in England, who came to America 1639. Such naming of offspring seems to imitate what was convention in the old country, where it may have diminished. Edison (talk) 03:08, 24 September 2012 (UTC)
- Thanks--I had wondered if it might be a relic from ye olde Britain. I've never come across it in historic British contexts, but perhaps I need to read more old genealogies. HenryFlower 04:56, 24 September 2012 (UTC)
- The linked source seems to have been written in the US in 1950. The same chap, Herbet Pelham, is described in the 1904 Lincolnshire Pedigrees as "Herbert Pelham of Swinehead". No mention of any Roman numerals for his entry in the Dictionary of National Biography either. The only Google results for "Herbert Pelham III" are in American (and one Norwegian!) publications. Alansplodge (talk) 02:09, 25 September 2012 (UTC)Alansplodge (talk) 01:52, 25 September 2012 (UTC)
- Thanks--I had wondered if it might be a relic from ye olde Britain. I've never come across it in historic British contexts, but perhaps I need to read more old genealogies. HenryFlower 04:56, 24 September 2012 (UTC)
- But in Britain, any given title refers to only one living peer. It's only when there's a need for disambiguation from dead peers that enumeration comes into play. Among the living, those in line for a peerage title take subordinate courtesy titles to distinguish themselves from their father and grandfather. So the eldest son of the Duke of Wellington would be known as the Marquess Douro during his father's lifetime; his son would be the Earl of Mornington, and his son Viscount Wellesley. In American families who repeatedly use the same name and wind up with I, II, III, etc., it may be worth mentioning that III might well be known as "Tripp" at the country club. - Nunh-huh 10:36, 24 September 2012 (UTC)
- If the Howells were a poor farming family who named, say, the oldest son after the father in each generation, then Thurston Howell III might be a man of no property or influence, and would not be a subject of ridicule in the US. I have seen such things many times in genealogy research. It is a custom brought from England, like "Herbert Pelham III," born 1600 in England, who came to America 1639. Such naming of offspring seems to imitate what was convention in the old country, where it may have diminished. Edison (talk) 03:08, 24 September 2012 (UTC)
- In Britain there are peers, who use titles rather than their given/surnames, and most titles come with numbers. The 19th Viscount Gibbetworthy is quickly differentiated from any of his predecessors. Same for ruling monarchs. But people of the lower orders would have been seen to be somewhat pretentious, and getting a bit beyond their station, if they adopted such a practice. In the US, where there is officially no institutionalised class system, I suspect it's only those who've become public figures, or leaders in a certain field, or have inherited wealth and fame and are expected to continue the dynasty, or see themselves as apart from the common herd, who would do this. Would Thurston Howell, III have had the ordinal if he were a nondescript nonentity such as a file clerk on average wages? Hardly. (No offence to nondescript nonentities, or file clerks on average wages.) -- ♬ Jack of Oz ♬ [your turn] 02:10, 24 September 2012 (UTC)
- See also Alexander Monro (secundus) and Alexander Monro (tertius).
- Quintus, Sextus, Septimus, Octavius and Decimus are also recorded as given names, but they are either an indicator of birth order within the same set of siblings, or maybe the holder was named after an earlier famous Quintus, Septimus etc. -- ♬ Jack of Oz ♬ [your turn] 02:44, 25 September 2012 (UTC)
- Re: your question about whether this enumeration is done elsewhere, it is the practice in traditional theater of Japan for a master performer when retiring to hand down their name to an apprentice or son, none the worse for wear, only with a higher numeral. A brief google search gave me this Kabuki related page [3] where there is mention of 'the first name changing of the 21st century' and some minor scandal about it. The father, Bandō Mitsugoro IX handed the name over to the son, making him Bando Mitsugoro X. Clicking on the link for this name in the article gives a CV that reads almost like a thoroughbred breeding history: [4].
- Also, staying in Japan, similarly to what Jack mentioned above, traditionally siblings would be given numbered names, with the names Taro, Jiro, Saburo, Shiro, Goro roughly corresponding to first son, second son and so on to fifth son. 164.71.1.221 (talk) 03:51, 25 September 2012 (UTC)
Capital punishment
Can someone tell me the moral justification for a state killing someone for killing someone? It seems duplicitous to me. What am I missing? --Anthonyhcole (talk) 13:08, 23 September 2012 (UTC)
- Permanent removal. Also allegedly a deterrent, but the jury's still out on that. ←Baseball Bugs What's up, Doc? carrots→ 13:16, 23 September 2012 (UTC)
- In addition to a deterrent and preventing future crimes, there's also the element of retribution: an eye for an eye. A sense of justice. If you think capital punishment is equivalent to murder, then imposing a fine is equivalent to theft, and arresting someone is equivalent to kidnapping. - Lindert (talk) 14:04, 23 September 2012 (UTC)
- Wow, for once an interesting analogy I haven't heard before! μηδείς (talk) 00:29, 24 September 2012 (UTC)
- I agree, 100%. Great analogy! Joseph A. Spadaro (talk) 20:57, 29 September 2012 (UTC)
- Punishments generally have up to four purposes: retribution, deterrent, prevention and rehabilitation. Obviously, capital punishment doesn't do the last one (a lot of punishments only to two or three of them) but the arguments for the other three are basically the same as for any other punishment. The counter-arguments are also pretty much the same as for other punishments. You can argue that it doesn't actually achieve the desired result (the evidence for capital punishment being an effective deterrent is pretty weak, for example) and you can argue that the punishment is disproportionate (ie. you can argue that the immorality of executing someone outweighs the benefits - some people put infinite weight to that immorality, so would never accept capital punishment regardless of the benefits). You can also argue that the punishment does more harm than good (eg. putting people in prison exposes them to other criminals and get them involved in more serious crimes once they are released), although I'm not sure you could construct an argument against capital punishment along those lines (except in the case of martyrs). --Tango (talk) 14:30, 23 September 2012 (UTC)
- Why don't they just greatly restrict inmates' fraternization? Sure there's no space, but so many of them are there just for being drug addicts anyway, release them to rehab and you might get it back. And why do guards still work in a place where they can get shanked and prisoners who are utterly self-serving on the outside keep living there (Jeffrey Dahmer was killed in prison) when to enter solitary/protective custody all they have to do is ask. Never made sense to me why they give them sharpenable things and put so many of them together at once to fight and trade crime tips when the crazy are put in a rubber room. 96.246.70.87 (talk) 16:09, 23 September 2012 (UTC)
- I agree. I'd certainly isolate them completely from other criminals. Just as "associating with known criminals" is bad for rehabilitation after release, it's also bad while still in custody. There was one prison that would put people in isolation with nothing but a Bible, but that seemed to drive some insane. I'd also give them educational materials, sessions with counselors (perhaps by CCTV or over the Internet), videos on anger management, etc. StuRat (talk) 18:21, 23 September 2012 (UTC)
- Lack of human contact can have serious psychological effects, even with things to keep you occupied. You also risk them losing what social skills they have, which won't serve them well on the outside. --Tango (talk) 19:01, 23 September 2012 (UTC)
- The interaction with counselors should handle this (I don't think the human interaction necessarily needs to be face-to-face). StuRat (talk) 21:46, 23 September 2012 (UTC)
- [citation needed]. I don't think you know anything about what you're talking about, here. If you're interested in getting informed, this article is a pretty nice way to start. --Mr.98 (talk) 02:22, 24 September 2012 (UTC)
- And you apparently lack basic reading comprehension skills. As I've said twice now, they would have interactions with counselors. Being isolated from other prisoners is not the same as total isolation, which is what your link talks about. They would also have things to do, windows, etc. And, while babies need actual physical contact, adults can interact with each other remotely, like we are doing now. The goal is not to eliminate all human contact, only negative human contact. StuRat (talk) 02:42, 24 September 2012 (UTC)
- Per usual you are making things up, wholesale, as you go along. It is obvious you know not a thing about what you're talking about. You are worse than the participants on Yahoo! Answers, because at least they have the good grace to mix in enough misspellings that no random person on the Internet would ever confuse them for a reliable source. --Mr.98 (talk) 15:26, 24 September 2012 (UTC)
- And you apparently lack basic reading comprehension skills. As I've said twice now, they would have interactions with counselors. Being isolated from other prisoners is not the same as total isolation, which is what your link talks about. They would also have things to do, windows, etc. And, while babies need actual physical contact, adults can interact with each other remotely, like we are doing now. The goal is not to eliminate all human contact, only negative human contact. StuRat (talk) 02:42, 24 September 2012 (UTC)
- Are you both adults?Ptg93 (talk) 12:29, 24 September 2012 (UTC)
- It's hard to avoid giving them sharpenable things - toothbrushes are commonly sharpened and they can't do without them. I suppose you could make them hand in their toothbrush every time they use it, but it quickly becomes a logistical nightmare. It's also bad for morale to treat prisoners like that. --Tango (talk) 19:01, 23 September 2012 (UTC)
- A recent protest at an Australian prison was over the issuing of toothbrushes with rubbery handles. They were stiff enough for brushing one's teeth, but useless for sharpening into a weapon. HiLo48 (talk) 08:09, 24 September 2012 (UTC)
- Surely you know we have an article capital punishment with a lot of material related to your question, including a lot of pro and anti external links. 69.228.171.70 (talk) 15:36, 23 September 2012 (UTC)
Back to the original question... Any moral justification for capital punishment has to justify the fact that it's impossible to compensate the subject if they are wrongly convicted. HiLo48 (talk) 18:29, 23 September 2012 (UTC)
- I don't see how that's any different from anything else done (rightly or wrongly) in the name of the public interest, that could result in someone getting killed accidentally. A bad legislative decision about, say, health policy or starting a war, will probably cause a lot more unjustified deaths than all the incorrect capital punishment verdicts in a comparable time period put together. Overall though I think this question is in WP:NOTFORUM territory and the OP is better off checking the existing sources. 69.228.171.70 (talk) 21:34, 23 September 2012 (UTC)
- Capital punishment is not an accidental death. And anyway, I was more making the comparison with fines and imprisonment based on incorrect conviction. A fine can be simply repaid to the innocent person. Financial compensation and state apologies are typical for wrongful imprisonment. But there's no easy way to compensate a dead person for their death. HiLo48 (talk) 00:04, 24 September 2012 (UTC)
- Cole is by no means a newbie, so I wonder why he brought this up. Maybe he could tell us? ←Baseball Bugs What's up, Doc? carrots→ 21:40, 23 September 2012 (UTC)
- The question arose during a conversation over dinner. We looked at the Capital punishment in the United States article and didn't see the answer there. I was hoping someone might be able to point me to a version of how a state reconciles a prohibition against killing with the act of retributive killing, they seem incompatible. This is a topic I know nothing about and thought if there was a well-known, boilerplate-type standard justification that states typically wheel out, someone might point me to it. Please don't go to any trouble if you don't know the answer; I just raised it in case somebody here knew where to look for the answer. I'll check out Capital punishment later. --Anthonyhcole (talk) 06:19, 24 September 2012 (UTC)
- I imagine that for many American conservatives the justification is in Leviticus 24:17 - "Whoever takes a human life shall surely be put to death." HiLo48 (talk) 08:16, 24 September 2012 (UTC)
- And maybe more importantly, because it is outside the context of the Mosaic law, and thus not limited to Israel, Genesis 9:6: "Whoever sheds man’s blood, by man his blood shall be shed, for in the image of God He made man.".
- I don't see how they make the logical jump from "We are made in God's image" to "therefore, murderers must be executed". Death penalty opponents would argue that this leads to exactly the opposite conclusion. StuRat (talk) 10:13, 24 September 2012 (UTC)
- Ehm, have you read the first part of the sentence? It's not conservatives that reach that conclusion, it's the author of Genesis (or rather, God Himself quoted by the author of Genesis). - Lindert (talk) 10:31, 24 September 2012 (UTC)
- Yes, by "they", I mean "the authors of the Bible". It seems an illogical conclusion, even if you accept the premise. StuRat (talk) 10:33, 24 September 2012 (UTC)
- The arguments I hear are not really about "moral" justification, but rather "social" justification - namely, that society has the right to permanently protect itself from a murderer, and the only certain way to do that is by exterminating him in a method prescribed by law. Keep in mind that the Ten Commandments proscription is against murder, which is the unlawful taking of life. ←Baseball Bugs What's up, Doc? carrots→ 06:06, 26 September 2012 (UTC)
Critique of Jared Diamond
There is a discussion going on on the Science desk at the moment (that I think should probably be here, which is why I'm asking my question here) about the theories of Jared Diamond and how a lot of people disagree with them. I have read Guns, Germs, and Steel and found it a very interesting and plausible theory. Can anyone recommend a good, detailed critique of it, ideally also written for the layman? --Tango (talk) 14:55, 23 September 2012 (UTC)
- Googling "Jared Diamond Critique" gave me a couple right off the bat here and here. These are opinion pieces, and I have no idea how reliable these folks are, but they are what you ask for, I think. Mingmingla (talk) 15:59, 23 September 2012 (UTC)
- I'm not sure "posting the first blog posts that came up while Googling" is what Tango meant by "recommend[ing] a good, detailed critique of it." --Mr.98 (talk) 18:11, 23 September 2012 (UTC)
- Indeed - I can find critiques myself easily enough, but I know refdeskers tend to have quite good judgement about how good references are, so I'm hoping to make use of that to find something actually worth reading. --Tango (talk) 19:03, 23 September 2012 (UTC)
- Did you check Guns, Germs, and Steel#Criticism? 69.228.171.70 (talk) 21:56, 23 September 2012 (UTC)
- Indeed - I can find critiques myself easily enough, but I know refdeskers tend to have quite good judgement about how good references are, so I'm hoping to make use of that to find something actually worth reading. --Tango (talk) 19:03, 23 September 2012 (UTC)
- I'm not sure "posting the first blog posts that came up while Googling" is what Tango meant by "recommend[ing] a good, detailed critique of it." --Mr.98 (talk) 18:11, 23 September 2012 (UTC)
- There's a rather important review article in the AHR: Gale Stokes "The Fates of Human Societies: A Review of Recent Macrohistories" The American Historical Review, Vol. 106, No. 2 (Apr., 2001), pp. 508-525: "one might ask if it is even possible to write scholarly work on a macrohistorical scale?" I'd suggest that this review dismissed Diamond by ignoring him; Diamond is used in the review as a controversialist with a narrow causative vision (geographical determinism) and a decent enough question poorly answered. Fifelfoo (talk) 04:56, 24 September 2012 (UTC)
Laudabiliter
Just a few questions:
- Were the Popes elected after Adrian IV always in favour of the Laudabiliter?
- Was the purpose of the Laudabiliter to allow the English to conquer Ireland and convert the "heretics" (as someone had told me), or was it just an excuse and instead used to impose territorial gains over the island?
- If the latter is true, were the States of the Church and the subsequent Popes aware of this "scandal"?
- Could it have been possible to denounce the Laudabiliter, even in the 1460s? How would the procedure be accomplished?
Thanks. 50.101.203.150 (talk) 17:00, 23 September 2012 (UTC)
- As you are likely aware, the situation is complex. Firstly, I'm not sure that we can know the position of every Pope on the matter, at some point in history the matter became moot because of other historical developments. For example, I don't imagine John Paul II gave the matter much thought at all, though he was certainly keenly aware of contemporary Irish-English relationships; I don't know that he thought on the matter in terms of an 850-year old bull. Secondly, the actual authenticity of the bull has been questioned repeatedly throughout history, according to our article, almost as soon as it was issued. Many popes after Adrian IV, however, continued to refer to it and cite it as though they treated it as legitimate. Thirdly, the Pope was a secular ruler as much as a leader of the Catholic Church, especially so in the middle ages, and he was not immune to political intrigue and manouvering. The fact that Adrian IV was English is not insiginficant in the history of the Bull, and like all politics, the status of the bull is subject to realpolitik, as was the rest of Papal actions throughout history. For example, our article cites a bull by Paul IV confirming Henry VIII's daughter Mary as legitimate Queen of Ireland (and not Henry VIII) under the terms of the original Bull, over Henry's excommunication and apostasy. --Jayron32 17:35, 23 September 2012 (UTC)
could i pay for internet tv like satellite tv
i have a fast broadband connection (30 megabit or so) and a big lcd monitor and high-quality soundcard. i'm living in a different country, but i know some people pay for international satellite tv to use on their tv. i don't have a tv proper, but can't I pay for 300 international channels all via internet or IP (irrespective of where I'm living)? If it's only available in certain geographic areas I could also pay for a vpn. I'm interested in channels from America. but I'm also curious now if I could do the same thing for France, Germany, Italy, Spain, and the UK. Thanks. 80.98.245.172 (talk) 19:25, 23 September 2012 (UTC)
- You would have to get that from your Internet Service Provider. Yours is Upc Magyarorszag Kft.. Their web page seems to show that they provide digital television, so you should contact them. Looie496 (talk) 19:41, 23 September 2012 (UTC)
- Something else you might want to consider is a tuner card and TV antenna. This allows you to watch broadcast TV, for free (after the initial purchase), on your computer. And, unlike Internet TV, it puts a minimal load on your computer, so you can have the TV going in one window and use the computer in another, without each slowing the other down. StuRat (talk) 20:06, 23 September 2012 (UTC)
- Also note that a large amount of TV is available over the Internet for free (with ads). See Hulu.com, for example. Some programs have their own web sites, too, like SouthParkStudios.com. StuRat (talk) 20:09, 23 September 2012 (UTC)
- A lot (as in a very large percentage) of this is US-only due to licensing and copyright issues. You can't even get Hulu in Canada, let alone in Hungary. --NellieBly (talk) 21:17, 23 September 2012 (UTC)
- The trouble is that program makers still sell licences to show their content to broadcasters on a regional basis. So the rights to show say Formula 1 are owned by different broadcasters in different countries. Recently those content contracts have often (but not at all always) been extended to allow the same broadcasters to show the content on their internet channels - but the geographic restriction remains the same, so those broadcasters are contractually mandated to limit visitors to IPs that geolocate within their territory. The two things StuRat cites as examples both do this: neither is available in the UK (the latter has a similar but different service that covers the UK). So Hulu isn't available outside the US, the BBC's iplayer (at least its TV content) isn't available outside the UK, and the Norwegian Broadcasting Corporation's internet feed isn't available outside Norway. Some broadcasters (particularly those like the BBC and NRK) which also create lots of their own content may chose to allow that to be viewed everywhere (NRK's is here), but you tend to get the kind of stuff that foreigners wouldn't buy anyway (brilliant though Supernytt is). So, as you say, you can use a VPN to make yourself appear in a location, which allows a provider to feel they can send you the content. Probably the broadcasters should be blocking VPNs, and I expect if their use increases the content providers will force them to. Apparently Americans were using VPNs to subvert NBC's restrictive, time-delayed coverage and watch BBC live instead [5] - this apparently didn't hurt NBCs ratings, but if it did they'd surely be shrieking at the BBC, IOC, and VPN companies, so as to protect that content they paid billions for. This problem already exists for satellite - for example, coverage of English football is sold to different European broadcasters who transmit it to their own countries on satellites - but the satellites cover much of western Europe, so those companies have to protect the content with encryption and only sell the decryption cards within their jurisdiction. This was the substance of a recent legal dispute in the UK (story) where a pub bought decoding equipment (and a subscription) in Greece (where the content was cheaper than in the UK) and then showed it in her UK premises. It's possible that the fallout of that case may mean the entire EU becomes a single licensing area (because, if that case is upheld generally) it could be illegal to prevent someone in one EU country viewing the country-limited content intended for another EU member state. In the meantime, content is still licensed nationally, so I think you're stuck with using VPNs to subvert that. -- Finlay McWalterჷTalk 21:24, 23 September 2012 (UTC)
Newfoundland Act 1949
Why doesn't the Newfoundland Act have a year in its name? Did Parliament forget to append a year to the name when they changed it in 1982? Newfoundland Act 1949 and Newfoundland Act, 1949 are both redlinks, and I hesitate to create them if there be a good reason for the lack of a year. Nyttend (talk) 21:00, 23 September 2012 (UTC)
- Acts usually have a year in their legal name to differentiate them from earlier acts of the same name. If the Newfoundland Act of 1949 is the first act of that name introduced in the Parliament of the United Kingdom, it would not contain a year in its title. Note that some acts have years in their common name that don't exist in their legal name - the Sophia Naturalization Act 1705 is legally the "Act for the Naturalization of the Most Excellent Princess Sophia, Electress and Duchess Dowager of Hanover, and the Issue of her Body". Edited to add: if people are (mistakenly) calling the "Newfoundland Act" the "Newfoundland Act 1949", that's a good reason to have a redirect even if the usage is wrong; redirects don't exist to justify incorrect usage but to allow readers to find articles. We have (or at least should have) a redirect to American Broadcasting Company at American Broadcasting Corporation specifically because people make that mistake. --NellieBly (talk) 21:24, 23 September 2012 (UTC)
- Really? Short title doesn't say anything about the year being included only if previous acts had the same title; the only thing that it says about the omission of a year is in Canadian federal legislation. It also refers to the "Artisans and Labourers Dwellings Act (1868) Amendment Act (1879) Amendment Act 1880", and if a year were omitted in the title of the first act by a certain name, this would surely be it; I can't imagine that there were multiple candidates for the title "Artisans and Labourers Dwellings Act (1868) Amendment Act (1879) Amendment Act". Meanwhile, on your Sophia Naturalization Act, this is likely the only act with this title, and anyway that title was presumably conferred by one of the Short Titles Acts, as short titles apparently weren't used in 1705. Nyttend (talk) 01:44, 24 September 2012 (UTC)
- It was originally titled the British North America Act 1949, but was renamed in Canada on the patriation of the Canadian Constitution from the United Kingdom in 1982. -- ♬ Jack of Oz ♬ [your turn] 02:50, 24 September 2012 (UTC)
- I previously missed the "in Canada" from the passage that you quote. So is it officially the "British North America Act 1949" in the UK to the present day? I don't know how to search for British laws. Nyttend (talk) 04:56, 24 September 2012 (UTC)
- It was originally titled the British North America Act 1949, but was renamed in Canada on the patriation of the Canadian Constitution from the United Kingdom in 1982. -- ♬ Jack of Oz ♬ [your turn] 02:50, 24 September 2012 (UTC)
United States Politicians With Notability in Other Fields
Which U.S. politicians were notable for something else (besides being military men/women) before getting into politics, besides Al Franken, Jon Runyan, and J. C. Watts? I would prefer that modern U.S. politicians be given in the answers, but you can also talk about historical U.S. politicians (besides those who were famous as military men). Futurist110 (talk) 21:56, 23 September 2012 (UTC)
- There's a whole category at Category:American athlete–politicians. 69.62.243.48 (talk) 22:18, 23 September 2012 (UTC)
- (ec) Yeah, a lot of actors come to mind, maybe as many as military. Ronald Reagan, Sonny Bono, Clint Eastwood, etc. Maybe so many to exclude them like military. John Glenn and Harrison Schmitt were both astronauts (Glenn a Navy pilot, Schmitt a geologist). Mitt Romney ran an investment firm and is a clergyman. Bill Bradley was a pro basketball player. Michael Bloomberg was/is a media mogul. There's lots more and might be a good "list" article. 69.228.171.70 (talk) 22:24, 23 September 2012 (UTC)
- Romney was not a notable clergyman, as far as I know. Ptg93 (talk) —Preceding undated comment added 23:09, 23 September 2012 (UTC)
- Romney's role as a Mormon cleric has gotten quite a bit of media attention recently[6] but ok, it might not have been notable if he hadn't become famous for other reasons. 69.228.171.70 (talk) 00:20, 24 September 2012 (UTC)
- Romney was not a notable clergyman, as far as I know. Ptg93 (talk) —Preceding undated comment added 23:09, 23 September 2012 (UTC)
Dwight D. Eisenhower?(oops, I see you said apart from the military) Benjamin Franklin? Steven Chu? 81.98.43.107 (talk) 23:24, 23 September 2012 (UTC)- He said it twice. Maybe you should be using double strike. :) -- ♬ Jack of Oz ♬ [your turn] 23:44, 23 September 2012 (UTC)
- Jack Kemp was a starting QB for the Buffalo Bills back in the old AFL before a lengthy political career. Heath Schuler is a more recent football transplant to politics. Ben L. Jones played Cooter on the Dukes of Hazzard before being elected to the House of Representatives. Fred Thompson is yet another notable actor who became a politician. --Jayron32 00:12, 24 September 2012 (UTC)
- Woodrow Wilson was a professor of jurisprudence, writer and educator, rising to become President of Princeton University. -- ♬ Jack of Oz ♬ [your turn] 00:22, 24 September 2012 (UTC)
- Sen. Jim Bunning (R–Kentucky) pitched a no-hitter for the Phillies in 1964. (Also, Ben Franklin was not really a politician, as he never held any elected office.) → Michael J Ⓣ Ⓒ Ⓜ 01:21, 24 September 2012 (UTC)
- You don't have to have to be elected to be considered a politician. His work in France was politicking of the highest order. Besides, he was elected President of Pennsylvania. Clarityfiend (talk) 01:40, 24 September 2012 (UTC)
- And Franklin held elected offices besides that one. He was elected to and served in the Second Continental Congress. --Jayron32 02:26, 24 September 2012 (UTC)
- A large number of people who rose to prominence after the Civil War were probably notable by their military service, as they would have been bound to be written up in the papers. I was thinking though of James A. Garfield's mathematical exploits ...--Wehwalt (talk) 02:36, 24 September 2012 (UTC)
Thank you very much for all of your answers. I wonder if there was ever a prominent notable woman who then entered politics (whose notability was not associated with being family to a politician or with the military). Futurist110 (talk) 05:56, 24 September 2012 (UTC)
- I wonder if Madeleine Albright fits your criteria? She's certainly had a very interesting life. --TammyMoet (talk) 08:44, 24 September 2012 (UTC)
- Not US but Angela Merkel has published several papers due to her career in chemistry before politics. Dismas|(talk) 09:22, 24 September 2012 (UTC)
- Wasn't Margaret Thatcher also a working research chemist before entering political life? --Jayron32 18:42, 24 September 2012 (UTC)
- Those ladies were never notable in their pre-politics life, and it's unknowable whether they would ever have become notable had they remained in chemistry. -- ♬ Jack of Oz ♬ [your turn] 20:56, 24 September 2012 (UTC)
- Wasn't Margaret Thatcher also a working research chemist before entering political life? --Jayron32 18:42, 24 September 2012 (UTC)
- And again in the UK, Glenda Jackson was an actress before she entered Parliament. --TammyMoet (talk) 13:19, 24 September 2012 (UTC)
- Elizabeth Warren was a law professor, then ran a government consumer agency, and is now a US Senate candidate. Hillary Clinton was First Lady before entering politics on her own, served as a Senator and is now Secretary of State. Actress Roseanne Barr is currently a third-party candidate for US President. Carly Fiorina and Meg Whitman were both business executives before running for office (unsuccessfully, but they were serious candidates). In Sweden, Anna Troberg is a novelist who ran a publishing company, then got into a debate about copyright policy with members of the Swedish Pirate Party, that led to her joining the party and eventually becoming its leader. 69.228.171.70 (talk) 18:46, 24 September 2012 (UTC)
- Mary Bono was semi-notable as the third wife of the more notable Sonny, before entering politics. --Robert Keiden (talk) 00:11, 27 September 2012 (UTC)
- Among the males, I'd consider Teddy Roosevelt- (notable as historian, military "adventurist" (though never enlisted)) but he was active in politics from the very beginning. He was more broadly NOTABLE for non-political stuff, before 1900.--Robert Keiden (talk) 00:11, 27 September 2012 (UTC)
- Not US but Angela Merkel has published several papers due to her career in chemistry before politics. Dismas|(talk) 09:22, 24 September 2012 (UTC)
Full metal jacket, the film, and real training
How realistic is the training in the film Full metal jacket? Ptg93 (talk) 23:50, 23 September 2012 (UTC)
- I've thought about this myself. I do recall people on internet forums who served in Vietnam saying that Stanley Kubrick got the boot camp scenes mostly right, as per their own experiences... though in real life, wouldn't Pvt. Pyle have been let go after a couple of weeks or so when it became clear that he wasn't hacking it - long before he was driven to blowing his stack? --Kurt Shaped Box (talk) 23:59, 23 September 2012 (UTC)
- One thing I do recall being mentioned was that sometimes in the movie, the Privates would sometimes refer to themselves in the first person in front of the Drill Instructor ('I don't know...' instead of 'The Private does not know...' etc.) without him going crazy on them, as would have happened in real life back then (or maybe still does?). --Kurt Shaped Box (talk) 00:07, 24 September 2012 (UTC)
- Yes, it seems that if you are not physically fit enough, you won't go into basic military training with weapons and stuff. You'll go through the route of United_States_Army_Basic_Training#Reception_Battalion.~Maybe it was different at the time. That's another issue, maybe the film is realistic, but the army has changed, making it anachronistic. Ptg93 (talk) 00:19, 24 September 2012 (UTC)
- Keep in mind this was the Marines, not the Army, and regimentation may have been a tad more severe. I also think it unlikely that guys in basic training in any branch would be allowed free access to rifles and ammunition. But it does make for a dramatic story. ←Baseball Bugs What's up, Doc? carrots→ 03:26, 24 September 2012 (UTC)
- As an aside, regardless of how accurate it was on depicting the Marines, the film may have accurately portrayed what it was probably like on a Kubrick film set, as he was notorious for pyschologically and physically abusing his actors, often to the point of genuine injury or mental anguish. In A Clockwork Orange, Malcolm McDowell suffered eye damage and broken ribs because Kubrick wanted the scenes where they were suffered to be as realistic as possible, so McDowell was genuinely beaten and tortured. Almost every one of his films featured major actors breaking down after having to shoot the same scenes dozens upon dozens of times. He basically lied to George C. Scott on Dr. Strangelove so he could get a comic performance out of him, tricking him to think that his absurdist "warm up" scenes weren't being filmed. Shelly Duvall was physically ill from the stresses of working on The Shining, in many ways her panicked, hysterical performance wasn't as much acting as it was a natural result of having to work around Kubrick for extended periods of time. --Jayron32 03:58, 24 September 2012 (UTC)
- None of the above was wrong, because Kubrick could do no wrong. -- ♬ Jack of Oz ♬ [your turn] 04:09, 24 September 2012 (UTC)
- I saw a quote somewhere from R. Lee Ermy, the ill-fated drill instructor, who was asked about Kubrick and he described him as "a prince among men". Of course, that might be because Kubrick basically told the former real-live D.I., "OK, we're rolling. Do your thing." ←Baseball Bugs What's up, Doc? carrots→ 04:17, 24 September 2012 (UTC)
- Oh, some actors positively worshiped Kubrick. Jack Nicholson was among them. Of course, he's also batshit insane, so there is that. Ermy was, as you note, party to Kubrick's psychological abuse. Given that Kubrick's famously long filming schedule and penchant for wasting huge amounts of film on retake after retake, the actors in FMJ had to deal with literally weeks of unbroken ten-hour days of Ermy screaming in their faces. Plus there's the fact that he filmed much of the post-boot camp scenes on a highly-toxic hazardous waste site, several actors complained of ill-health for some time afterwards as a result of working long days in that environment. --Jayron32 05:35, 24 September 2012 (UTC)
- I saw a quote somewhere from R. Lee Ermy, the ill-fated drill instructor, who was asked about Kubrick and he described him as "a prince among men". Of course, that might be because Kubrick basically told the former real-live D.I., "OK, we're rolling. Do your thing." ←Baseball Bugs What's up, Doc? carrots→ 04:17, 24 September 2012 (UTC)
- Slim Pickens supposedly refused to work with Kubrick again after "Dr. Strangelove" unless unless there would be a contractual clause limiting Kubrick to a maximum of 100 takes per scene... AnonMoos (talk) 13:59, 24 September 2012 (UTC)
- A possible exaggerated story, though it does sound somewhat plausible in light of the other tales of Kubrick's love of retakes - the scene in A Clockwork Orange where the parole officer/social worker guy hocks a loogie right in McDowell's face from about 6 inches away was supposedly repeated 50+ times until Kubrick was perfectly happy that he'd got the arrangement of spit on his leading man's face just right in an aesthetic sense. Then he ordered a few more takes of it, just to be on the safe side. --Kurt Shaped Box (talk) 22:37, 24 September 2012 (UTC)
- None of the above was wrong, because Kubrick could do no wrong. -- ♬ Jack of Oz ♬ [your turn] 04:09, 24 September 2012 (UTC)
- As an aside, regardless of how accurate it was on depicting the Marines, the film may have accurately portrayed what it was probably like on a Kubrick film set, as he was notorious for pyschologically and physically abusing his actors, often to the point of genuine injury or mental anguish. In A Clockwork Orange, Malcolm McDowell suffered eye damage and broken ribs because Kubrick wanted the scenes where they were suffered to be as realistic as possible, so McDowell was genuinely beaten and tortured. Almost every one of his films featured major actors breaking down after having to shoot the same scenes dozens upon dozens of times. He basically lied to George C. Scott on Dr. Strangelove so he could get a comic performance out of him, tricking him to think that his absurdist "warm up" scenes weren't being filmed. Shelly Duvall was physically ill from the stresses of working on The Shining, in many ways her panicked, hysterical performance wasn't as much acting as it was a natural result of having to work around Kubrick for extended periods of time. --Jayron32 03:58, 24 September 2012 (UTC)
- I can supply that according to marines I know that were in the service around the time of Full Metal Jacket, R. Lee Ermy 100% captures the real drill instructor experience (which makes sense because I believe was a DI trainer). Shadowjams (talk) 03:19, 25 September 2012 (UTC)
- Next time you see one of them, maybe you could ask if the scene with "Gomer Pyle" wielding a fully-loaded rifle in the barracks is realistically possible. I could imagine the Marines might make their recruits sleep with their rifles and all that sort of stuff. But allowing them free access to ammunition would be just begging for trouble. ←Baseball Bugs What's up, Doc? carrots→ 21:42, 25 September 2012 (UTC)
I went through Army basic training just a couple years before the movie was filmed. My drill sergeants used some of the same profane phrases that showed up in the movie, so the lingo apparently travels between the services. I had a couple buddies in the Marines at the same time and they tell me that my experience was like summer camp compared to theirs. Once you've been through basic, the movie version doesn't seem too bad, just a bit exaggerated. Nowadays they're not supposed to hit or touch you, but I imagine that was different a generation ago. I did have a drill sergeant lean against me, pushing me over while I was doing pushups and while he was cussing a blue streak. But this was weeks into boot camp, and by that time those sort of things don't phase you at all. I thought he was a very nice man even at the time. We were both just doing our jobs. Access to live ammunition was strictly controlled. I've always assumed that in the movie, Private Pyle smuggled live rounds off the rifle range or something, which is possible if you're determined to break the rules. —Kevin Myers 00:39, 26 September 2012 (UTC)
- Some of the stuff the DI was spouting in Full Metal Jacket was the exact same stuff said by the DI in An Officer and a Gentleman several years earlier, so there must be kind of a standard "catalog" of stuff the DI's all say. Some of it's so outrageous it might be hard to keep a straight face, but of course if you show that kind of reaction then you'll really be in trouble. ←Baseball Bugs What's up, Doc? carrots→ 06:00, 26 September 2012 (UTC)
September 24
Early voting and changing votes in the U.S
For the sake of the question, let's say a person decides to vote early for President Barack Obama. Then, Mitt Romney surprisingly does so well in the presidential debates that the person who voted early for Barack Obama decides he or she wants to change his or her vote. If the person voted for one candidate during early voting, is it possible for that person to cancel his or her first vote and change it for the other candidate just in time before November 6? If so, how? 71.98.171.202 (talk) 00:45, 24 September 2012 (UTC)
- First thing: there is no "In the U.S." for this question. Each state has its own voting laws and procedures, so there is no way to answer for the U.S., per se. That said, it is possible that all states that do early voting may treat this the same way. That being said, AFAIK, once a ballot is submitted it isn't refundable. If you submit your ballot at 8AM on "voting day" and change your mind by noon, you can't go back to the polling place and get your ballot back then either. --Jayron32 00:56, 24 September 2012 (UTC)
- I am tempted to suggest that in most states without voter ID laws you just go vote as two dead people to cancel out your mistaken earlier vote. But I won't. μηδείς (talk) 00:58, 24 September 2012 (UTC)
- Good idea since for starters that won't generally work properly in Maine and Nebraska which evidentally don't have voter ID laws [7] [8]. Nil Einne (talk) 06:23, 24 September 2012 (UTC)
- Good idea since for starters that won't generally work properly in Maine and Nebraska which evidentally don't have voter ID laws [7] [8]. Nil Einne (talk) 06:23, 24 September 2012 (UTC)
- I am tempted to suggest that in most states without voter ID laws you just go vote as two dead people to cancel out your mistaken earlier vote. But I won't. μηδείς (talk) 00:58, 24 September 2012 (UTC)
States that require photo ID
States that request photo ID
|
States that require non-photo ID
States with no voter ID law
|
]] There are many states that don't have any voter ID laws. When I recently voted in the primaries here in Vermont, all I had to do was supply my name so that they could mark it off their list. Dismas|(talk) 07:44, 24 September 2012 (UTC)
- There might be a way to do this if you mailed in your ballot and your name were on the return address. Though I doubt that any state is going to bother with it and they'd just say that once it's done, it's done. It opens up too many cans of worms. And expanding on Jayron's point, if I cast my ballot at 8AM and then go back later, there's no way of telling mine from anyone else's ballot since my name isn't on it. It's just another ballot in a box full of ballots with no way of tracing any of them back to me. Dismas|(talk) 01:16, 24 September 2012 (UTC)
- Ballots that are mailed in, either "early voting" or absentee, are sent in a double envelope. The outer one has all the address stuff. The inner one is either blank or marked with basic information, such as 2012 Jones County General Election. If the inner envelope gets any other markings on it, it is considered void. The outer envelope is destroyed as soon as it arrives at the county voting office, and the inner envelope remains sealed until it is counted on election day. Therefore there is no way to know which ballot to give back if someone wanted it back. → Michael J Ⓣ Ⓒ Ⓜ 01:30, 24 September 2012 (UTC)
There's actually a good reason why voters can't change their votes, other than the logical reasons that it's very tough to track down - extortion. We want to reduce the chances that another person threatened or extorted someone else into changing their vote. You cast a vote for X at 8 A.M. and then change it 4 hours later, it looks suspicious. --Activism1234 05:00, 24 September 2012 (UTC)
- Isn't it a secret ballot? How would an extortionist know who you voted for? HiLo48 (talk) 07:59, 24 September 2012 (UTC)
- Vote early and vote often works well in Texas. Thincat (talk) 08:17, 24 September 2012 (UTC)
- In fact, there is very little evidence of existing voter fraud in the US, and what exists appears slanted towards issues with registration (which can be caught prior to elections), not with voting itself. [9] [10] — Lomn 13:36, 24 September 2012 (UTC)
- That's a conflation of "voter impersonation" and "voter fraud", and a dismissal of unlawful registration as not a problem, since in fact such registrations are often not caught.--Wehwalt (talk) 13:42, 24 September 2012 (UTC)
- A postulation of "voter fraud is endemic" and 'evidence' of "we can't catch them, so they must be there" is not particularly persuasive. To the best of my knowledge, there is no credible evidence of widespread voter fraud in modern US federal elections, but there is plenty of evidence that suggests it is not [11]. — Lomn 14:03, 24 September 2012 (UTC)
- No, the more obvious area of potential corruption is allowing state governors with obvious conflict of interest, and/or their appointees, the right to rule on what hanging chads mean. HiLo48 (talk) 00:49, 25 September 2012 (UTC)
- No argument from me -- but obviously that's a very different sort of problem than the blithe assertions above regarding "vote early, vote often" and "just give them some dead guy's name". — Lomn 04:07, 25 September 2012 (UTC)
- No, the more obvious area of potential corruption is allowing state governors with obvious conflict of interest, and/or their appointees, the right to rule on what hanging chads mean. HiLo48 (talk) 00:49, 25 September 2012 (UTC)
- A postulation of "voter fraud is endemic" and 'evidence' of "we can't catch them, so they must be there" is not particularly persuasive. To the best of my knowledge, there is no credible evidence of widespread voter fraud in modern US federal elections, but there is plenty of evidence that suggests it is not [11]. — Lomn 14:03, 24 September 2012 (UTC)
- That's a conflation of "voter impersonation" and "voter fraud", and a dismissal of unlawful registration as not a problem, since in fact such registrations are often not caught.--Wehwalt (talk) 13:42, 24 September 2012 (UTC)
- In fact, there is very little evidence of existing voter fraud in the US, and what exists appears slanted towards issues with registration (which can be caught prior to elections), not with voting itself. [9] [10] — Lomn 13:36, 24 September 2012 (UTC)
I was wondering which of these two books best describe the role that the Roman Catholic Church and the Pope played during WWII? Thank you. Futurist110 (talk) 05:57, 24 September 2012 (UTC)
- This is a reference desk. Your question is beyond its scope. Gabbe (talk) 06:45, 24 September 2012 (UTC)
- Not necessarily. While we shouldn't necessarily provide our own opinion, we could provide links to reliable critiques of said works. That is, our opinions don't matter, but some people's do, and we could link to those that do. --Jayron32 06:47, 24 September 2012 (UTC)
- Yeah, I'd like to see some reliable critiques of both of these. Futurist110 (talk) 07:15, 24 September 2012 (UTC)
- I think it's fair to characterize The Myth of Hitler's Pope as contrarian. Its publisher, Regnery Publishing, "specializes in conservative books characterized on their website as 'contrary to those of "mainstream" publishers in New York'" according to us. This suggests Hitler's Pope presents a more mainstream view, but that's hardly conclusive. Hitler's Pope appears to have been more widely reviewed, and and has a better article here. Contrarian opinions can be right, though. Why don't you tell us? Read them both, pay attention to their sourcing, and decide for yourself. --BDD (talk) 20:58, 25 September 2012 (UTC)
- Yeah, I'd like to see some reliable critiques of both of these. Futurist110 (talk) 07:15, 24 September 2012 (UTC)
- Not necessarily. While we shouldn't necessarily provide our own opinion, we could provide links to reliable critiques of said works. That is, our opinions don't matter, but some people's do, and we could link to those that do. --Jayron32 06:47, 24 September 2012 (UTC)
For term or traditional logician; Can term logic be sufficient to be the content of philosophy in dealing with central philosophical topics?
Logic is a very wide subject and is not only for tackling central philosophical issues as computer logic cannot conform to the field. Like wise if the concern is pure logic it cannot be directed to any other field for it is now a subject by itself, pure logic cannot be purely for philosophy. Thus to make logic more inclined to the needs of a certain field, say, philosophy, it should be scrutinized to be specific for the needs of philosophy, which is forming valid propositions, logical distinction and definition of terms, logical opposition, valid argumentation/ syllogism. So is it plausible? — Preceding unsigned comment added by Rt56h3 (talk • contribs) 11:49, 24 September 2012 (UTC) Before I can even atttempt to address this please define what you mean by logic and philosophy because it is in no way clear from your posts. Hotclaws (talk) 18:20, 26 September 2012 (UTC)
- You have asked a few of these questions over the past few days, and I don't think the Reference Desk is really equipped to respond in the way you are looking for. Have you tried asking on a more philosphically focused forum? Livewireo (talk) 14:54, 24 September 2012 (UTC)
Interpreting the old testament
When confronted with brutal passages of the Old Testament, some defending death penalty and condemning homosexuality, some Christians claim it is not really meant like that anymore, due to the change of times and such. Is this interpretation honest? Is it based on some recommendation in the New Testament? Ptg93 (talk) 12:37, 24 September 2012 (UTC)
- Like all theological questions, this has been debated to high heaven (no pun intended). The arguments and citations on this site are consistent with those who argue that Jesus' offer more or less undid all of the Old Testament laws. There are those who use a similar combination of cherry-picked and selectively-interpreted lines to come to the exact opposite conclusion. My basic anecdotal observation is that almost all modern Christians seem to prefer the former interpretation, because it gets you out of things like dietary prohibitions and stoning adulterers and other things that modern Christians consider to be, well, uncivilized and awful and generally inconsistent with the mode to salvation put forward by Christ. --Mr.98 (talk) 12:44, 24 September 2012 (UTC)
- And are there any impositions in the New Testament which collide with modern western values? Like stoning, amputations as punishment, homophobia, and so on. Ptg93 (talk) 13:29, 24 September 2012 (UTC)
- The traditional Christian view is that the purely "ceremonial" or "ritual" requirements of the Old Testament are suspended for Christians, but the "moral" or "ethical" requirements remain in force. However, even when Christians still regard things as wrong which the Old Testament condemns as wrong, that doesn't necessarily mean that Christians feel bound to impose the same penalties laid down in the Old Testament. That's the framework within which such matters have been debated. As for the New Testament, it doesn't lay down a legal code, and the example of Jesus and the woman taken in adultery has pretty much eliminated stoning as a punishment in Christian-majority societies... AnonMoos (talk) 13:52, 24 September 2012 (UTC)
- I would perhaps take issue with the implication here that Christian-majority societies have been someway "softened" in their religious punishments because of the New Testament recommendations. Historically this isn't very clear. Western societies as a whole have been marked by a graduate retreat from religious prosecution and cruel punishments, but this is a relatively recent development compared to the spread of Christianity, and arguably has more to do with the movement of the Christian world out of secular power than anything else. --Mr.98 (talk) 15:17, 24 September 2012 (UTC)
- I said only that in most cases, Christian societies haven't felt obliged to implement Old Testament punishments to the letter, and that stoning hasn't been an official judicial punishment. This says very little about overall level of cruelty... AnonMoos (talk) 23:50, 24 September 2012 (UTC)
- Certainly, particularly in the Epistles (see, for example, 1 Cor 14:34-35, women should keep silent in church and only ask about stuff once they're back home). As with the above, there have been countless debates about whether various injunctions were written for the specific original audience as guidance, for all Christians as requirements, or for some position between the two. — Lomn 13:55, 24 September 2012 (UTC)
- Some Christians go with a more literal interpretation of the Old Testament. For instance http://www.conservapedia.com/Woman_caught_in_adultery dismisses the bit about Jesus and the women caught in adultery as just liberal hogwash added later to the gospel. Dmcq (talk) 15:17, 24 September 2012 (UTC)
- Scripture is the inspired word of God and the Bible is infallible. Conveniently, however, We The Right People get to decide what is Scripture, and what not. I bet there is more biblical scholarship in The Brick Testment than in all of Conservapedia... --Stephan Schulz (talk) 16:03, 24 September 2012 (UTC)
- If you had cared to check the reference, you'd have noticed that it is from Bart Ehrman, who is an outstanding scholar and anything but conservative (he's an agnostic btw). In fact, if you look at the wikipedia article Jesus and the woman taken in adultery, you'll discover that the manuscript evidence is indeed against the passage being original (all manuscripts before the 5th century exclude it). - Lindert (talk) 16:22, 24 September 2012 (UTC)
- But Ehrman does not hold the position that the Bible is inspired and/or infallible. And, of course, the fact that it is a later addition to John in no way necessarily implies that it is more or less true than John. It's completely within God's (if you beliefe in that thing) power to have his wisdom converge from different sources.... --Stephan Schulz (talk) 16:46, 24 September 2012 (UTC)
- If you had cared to check the reference, you'd have noticed that it is from Bart Ehrman, who is an outstanding scholar and anything but conservative (he's an agnostic btw). In fact, if you look at the wikipedia article Jesus and the woman taken in adultery, you'll discover that the manuscript evidence is indeed against the passage being original (all manuscripts before the 5th century exclude it). - Lindert (talk) 16:22, 24 September 2012 (UTC)
- Scripture is the inspired word of God and the Bible is infallible. Conveniently, however, We The Right People get to decide what is Scripture, and what not. I bet there is more biblical scholarship in The Brick Testment than in all of Conservapedia... --Stephan Schulz (talk) 16:03, 24 September 2012 (UTC)
- Some Christians go with a more literal interpretation of the Old Testament. For instance http://www.conservapedia.com/Woman_caught_in_adultery dismisses the bit about Jesus and the women caught in adultery as just liberal hogwash added later to the gospel. Dmcq (talk) 15:17, 24 September 2012 (UTC)
- Certainly, particularly in the Epistles (see, for example, 1 Cor 14:34-35, women should keep silent in church and only ask about stuff once they're back home). As with the above, there have been countless debates about whether various injunctions were written for the specific original audience as guidance, for all Christians as requirements, or for some position between the two. — Lomn 13:55, 24 September 2012 (UTC)
- Lindert -- The "pericope" of Jesus and the woman taken in adultery did not originally belong to the one of the four canonical gospels, but that's not the same thing as saying that it's not rather early... AnonMoos (talk) 02:44, 25 September 2012 (UTC)
- Thanks very much for that reference to The Brick Testament, Revelation is far better than anything at Trapped in the Closet (South Park) ;-) Dmcq (talk) 17:27, 24 September 2012 (UTC)
- I suppose this depends on how you look at the Bible. Jesus at no point said that he had come to undo the Old Testament, but rather, Jesus is the fulfilment of it. (To grossly simplify, this is why the Old Testament is still relevant to Christians, without it, Jesus is just 'some guy', but with the Old Testament, he can become the promised 'Messiah'.) However, there are some passages in the New Testament which go against some of the passages in the Old. Examples being:
- Jesus stating that Man wasn't created for the Sabbath, but the Sabbath for man (which I would read as meaning that the prohibition on working on the Sabbath is only applicable so long as it benefits man)
- St. Paul wrote that circumcision wasn't necessary, so long as all the other commandments are followed.
- Jesus stated that nothing that you eat can make you unclean, only what you say can make you unclean.
- Jesus stating that forgiveness is more important that punishment. (You are to forgive not 7*7 times, but 7*77.)
- So, there certainly are some passages in the New Testament that would seem to negate passages in the Old. As for the rest, as others have pointed out, today, Christians focus more on the charity and forgiveness parts in the New Testament, rather than the stonings in the Old. Homosexuality, though, is even condemned in the New Testament, and arguments to legalise gay marriage within the framework of the church is more esoteric than 'Jesus invalidated the Old Testament'. V85 (talk) 17:00, 24 September 2012 (UTC)
- What do you mean by "it is not really meant like that anymore"? Meant by whom? The people who wrote the Old Testament are long dead, so they certainly can't "mean" anything today. As for Christians themselves, they've proven themselves capable of picking and choosing from scripture to support whatever belief they want--from under-the-radar pacifism under the Roman Empire, to corrupt tyrannical theocracies ruled by the Roman Catholic Church, to Westphalian sovereign states, to genocidal colonialism, to Nazism/Communism, to modern secular democracy with Enlightenment ideals.
- If you really meant "did the Old Testament's authors really mean to say what they did?", it would be disingenuous to suggest otherwise because of modern values. How can you possibly expect people in ancient Judah and Israel, separated from us by more than 2500 years, to have adopted the ideals of social movements that are less than 100 years old? There is no historical, cultural, or scientific reason to suspect that the authors believed anything other than what they claimed to. It's nonsensical to criticize the Ten Plagues for violating the laws of war, violating human rights, or harming civilians, because there was no concept of "laws of war", no concept of universal human rights, and no concept of protecting enemy civilians. It's nonsensical to criticize ancient Israel for not being democratic, because there was no concept of democracy or popular sovereignty, and there wouldn't be for another 2000 years. It's stupid to criticize Corinths or Timothy for being sexist and homophobic respectively, because feminism wouldn't become a major cultural force until the 1900's and LGBT wouldn't become mainstream until the 21st century. --140.180.242.9 (talk) 17:15, 24 September 2012 (UTC)
- One thing to note, the Bible only condemns sexual relations between homosexuals, not homosexuality itself. I welcome any verse that point to the contrary. Plasmic Physics (talk) 00:02, 25 September 2012 (UTC)
- Similarly, I only condemn the act of murder, not the thought of wanting to murder somebody. Prohibiting a thought is a futile undertaking until mind reading is invented. --140.180.242.9 (talk) 01:20, 25 September 2012 (UTC)
- Condemnation and prohibition are not equivalent - the Bible condemns hate-induced thoughts of murder, but does not prohibit it. This is why repentance is crucial, to willingly stop and turn away from sin. Plasmic Physics (talk) 01:48, 25 September 2012 (UTC)
- Similarly, I only condemn the act of murder, not the thought of wanting to murder somebody. Prohibiting a thought is a futile undertaking until mind reading is invented. --140.180.242.9 (talk) 01:20, 25 September 2012 (UTC)
- This is controversial, and not my field, but one idea is that there is a difference between the "Old Covenant", the arrangement of Abraham to make his seed "as numerous as the dust of the earth", and the "New Covenant", the teachings of Jesus by which men might receive eternal life. Things like a prohibition of homosexuality have a sort of sense to them if your goal is to have innumerable descendants. But if the goal is to be redeemed of sin in a perfect world, then universal love is the priority. Wnt (talk) 17:18, 24 September 2012 (UTC)
- Also, at least early Christians differentiated between the Noahide Laws, which applied to all humanity, and the Abrahamic covenant which e.g. requires circumcision and applies to Israel only. That was the argument to not demand circumcision and adherence to the Mosaic food laws from non-Jewish converts to Christianity. Of course, the Noahide laws forbid the consumption of blood (which is the substance of life and reserved to God), so black pudding is still out. --Stephan Schulz (talk) 18:02, 24 September 2012 (UTC)
- The prohibition on blood is repeated in the New Testament as well (Acts 15:29). --Trovatore (talk) 20:01, 24 September 2012 (UTC)
- So, who wins in the cage fight between Acts 15:29 and Matthew 15:1-19? More relevent is the different standard entirely as expressed in Romans 7, Romans 14, and 1 Corinthians chapters 8, 9, 10. That rather unambiguosly deals with how Christians should decide which behaviors they should and should not do. And it has nothing to do with any written law. --Jayron32 20:48, 24 September 2012 (UTC)
- The prohibition on blood is repeated in the New Testament as well (Acts 15:29). --Trovatore (talk) 20:01, 24 September 2012 (UTC)
- Also, at least early Christians differentiated between the Noahide Laws, which applied to all humanity, and the Abrahamic covenant which e.g. requires circumcision and applies to Israel only. That was the argument to not demand circumcision and adherence to the Mosaic food laws from non-Jewish converts to Christianity. Of course, the Noahide laws forbid the consumption of blood (which is the substance of life and reserved to God), so black pudding is still out. --Stephan Schulz (talk) 18:02, 24 September 2012 (UTC)
- One of the reasons I fell away from the "Christianity" in which I was brought up is that Christians can't agree on what it is. This thread reinforces my view. HiLo48 (talk) 23:42, 24 September 2012 (UTC)
- For my part, I wasn't saying anything about what Christianity is or isn't, or what it does or doesn't require in terms of the eating of blood. I just pointed out that, textually, the prohibition does appear in the NT as well.
- (By the way, drop by talk:mathematics some time and see whether mathematicians can agree on what mathematics is.) --Trovatore (talk) 00:20, 25 September 2012 (UTC)
- But I'm not likely to go to hell if I ignore the laws of mathematics. (That's if we can agree on what and where hell is, and whether or not it exists.) HiLo48 (talk) 00:55, 25 September 2012 (UTC)
Is Wikipedia really the best place to have an argument over Christianity... C'mon guys, no one's changing their views here, best to move it along. --Activism1234 00:57, 25 September 2012 (UTC)
Btw, if you're interested in learning more about the Judaic interpretation of the Old Testament (the only book common to all 3 Abrahamic faiths, not just Christianity), I'd recommend checking out the article Talmud, Midrash, and Mishnah. They don't help much for Christian interpretations though. --Activism1234 01:27, 25 September 2012 (UTC)
- Sorry, but there's no book which is "common to all 3 Abrahamic faiths" in the sense of being accepted as an authoritative scripture by all three. The traditional mainstream Muslim position is that the Qur'an and other Islamic books contain everything about religion that it is needful for ordinary believers to know, and if anything in Jewish or Christian scriptures contradicts anything in the Qur'an (even in the slightest detail), this automatically means that they are ipso facto "corrupt"... AnonMoos (talk) 02:54, 25 September 2012 (UTC)
For Islam, Tafsir and Esoteric interpretation of the Quran are good articles as well, as though don't apply to the Old Testament as much as to the Qur'an. --Activism1234 01:29, 25 September 2012 (UTC)
@StuRat, thanks for that revert, I didn't notice it (result of edit conflict). --Activism1234 01:36, 25 September 2012 (UTC)
Monogamy as part of culture
I was told that Punjabi culture or Sikh culture is or are the only culture(s) that practices monogamy meaning that they shouldn't divorce or do polygamy. Is there any other cultures that do that? — Preceding unsigned comment added by 70.53.229.120 (talk) 16:53, 24 September 2012 (UTC)
- But surely, that is also the ideal in the traditional, Christian view. Jesus forbid divorce (which is why Catholics get their marriages annulled), and one should also only have one wife. Is that how most Christians have lived during the last 2000 years? I don't know. Is it how most Christians live today? Possibly not. Is it an ideal aspired to by many? Absolutely. V85 (talk) 17:13, 24 September 2012 (UTC)
- Many, if not most, cultures throughout recorded history have held forms of monogamy as ideal. Are you referring to a specific set of cultures because Punjabi and Sikh cultures are hardly unique in their views of marriage? --Daniel 19:38, 24 September 2012 (UTC)
- I'd take a partial objection to that. Most societies have held female monogamy as an ideal. They've been rather inconsistent on the notion of male monogamy. Polygyny is far more common than polyandry, which is very rare indeed. Men, who historically have held most of the political power in societies, have decided they don't want their women running around on them, but they want to keep their own options open. --Jayron32 19:53, 24 September 2012 (UTC)
- Surely it would be possible for a society to have monogamy as the ideal, even if it allowed for polygamy. V85 (talk) 20:55, 24 September 2012 (UTC)
- While that may have been ture in ancient times, a lot of these same cultures have pretty much done away with polygamy for males, unless that's the nature of the country you live in (I know people in Lebanon where it's normal for them, while their brother in America would find it odd). As for divorce, there haven't been too many societies that say divorce, and only one I can think of is really Christianity. --Activism1234 23:49, 24 September 2012 (UTC)
- I didn't understand your last sentence, Activism1234. -- ♬ Jack of Oz ♬ [your turn] 02:11, 25 September 2012 (UTC)
- Epic hand spasm fail. I meant to write "haven't been too many societies that forbid divorce..." --Activism1234 22:23, 27 September 2012 (UTC)
- Thanks. -- Jack of Oz [Talk] 02:44, 28 September 2012 (UTC)
- Epic hand spasm fail. I meant to write "haven't been too many societies that forbid divorce..." --Activism1234 22:23, 27 September 2012 (UTC)
- I didn't understand your last sentence, Activism1234. -- ♬ Jack of Oz ♬ [your turn] 02:11, 25 September 2012 (UTC)
- I'd take a partial objection to that. Most societies have held female monogamy as an ideal. They've been rather inconsistent on the notion of male monogamy. Polygyny is far more common than polyandry, which is very rare indeed. Men, who historically have held most of the political power in societies, have decided they don't want their women running around on them, but they want to keep their own options open. --Jayron32 19:53, 24 September 2012 (UTC)
- Many, if not most, cultures throughout recorded history have held forms of monogamy as ideal. Are you referring to a specific set of cultures because Punjabi and Sikh cultures are hardly unique in their views of marriage? --Daniel 19:38, 24 September 2012 (UTC)
Gadsby copyright concern
O.K., so I'm readying an Amazon Kindle reissue of Gadsby, the 1939 novel Ernest Vincent Wright wrote without an "e". I've been working on it from the time I saw this Guy Kawasaki post on Google+. As part of the efforts, I've been busy with a biographical essay that will be included in this edition. (Almost 300 footnotes so far at this writing!)
I have a Kindle Direct Publishing (KDP) account, and part of the job there is to ensure whatever books I sell do not violate any copyrights. Gadsby is already in the U.S. public domain due to renewal failure (back in the late 1960s), and its copyright has already expired in countries observing 70 p.m.a or less.
But looking at the list of countries' copyright length, one thing bugs me:
Is the work really under copyright...in Spain?
It's worrying, because for authors who died in 1987 or earlier, Spain observes 80 p.m.a. according to the list. Wright died in 1939, so that would allow for a 2020 expiration. Then again, it's now a 70 p.m.a. since their laws changed in the 1990s.
KDP lets authors and publishers determine whether the book can be sold worldwide, or in a different set of countries, as conditions permit. For the latter option, there's a set of buttons that can turn on availability depending on the book's PD status.
With two weeks to go, I'd like to know: Should I turn on Spain's button or no? --Slgrandson (How's my egg-throwing coleslaw?) 18:00, 24 September 2012 (UTC)
- We are not permitted to give legal advice here, and I'm afraid this question falls into that category. Looie496 (talk) 18:27, 24 September 2012 (UTC)
- Where should I place it, then? --Slgrandson (How's my egg-throwing coleslaw?) 18:33, 24 September 2012 (UTC)
- At Wikipedia, the best place to ask copyright questions is at WP:MCQ. There are some knowledgable people (not lawyers, they're advice is useful but meaningless in a legal context) who frequent that board. --Jayron32 19:49, 24 September 2012 (UTC)
- Lawyers' advice are meaningless in a legal context?? The questioner needs a Spanish copyright lawyer (or someone, e.g. Amazon, who has access to one), not random people on the internet. --PalaceGuard008 (Talk) 10:13, 25 September 2012 (UTC)
- I'm pretty sure Jayron32 meant the advice of people on MCQ is useful but meaningless in a legal context because they're not lawyers (or if they are, they're definitely not providing legal advice). However I should add AFAIK the people at MCQ only really help with questions relating to wikipedia. They're not likely to help for questions which don't relate to wikipedia, particularly for real world cases that don't relate to wikipedia. I don't think Jayron32 intended to suggest otherwise but it's perhaps important to clarify. Nil Einne (talk) 10:28, 25 September 2012 (UTC)
- Yeah, it's not the lawyers who are useless in a legal context, its the MCQ people who are. I was unsure of the OPs original intent, but I recommended him to MCQ because at least some of the people there seem to have a grasp on copyright in some way. --Jayron32 12:43, 25 September 2012 (UTC)
- I'm pretty sure Jayron32 meant the advice of people on MCQ is useful but meaningless in a legal context because they're not lawyers (or if they are, they're definitely not providing legal advice). However I should add AFAIK the people at MCQ only really help with questions relating to wikipedia. They're not likely to help for questions which don't relate to wikipedia, particularly for real world cases that don't relate to wikipedia. I don't think Jayron32 intended to suggest otherwise but it's perhaps important to clarify. Nil Einne (talk) 10:28, 25 September 2012 (UTC)
- Lawyers' advice are meaningless in a legal context?? The questioner needs a Spanish copyright lawyer (or someone, e.g. Amazon, who has access to one), not random people on the internet. --PalaceGuard008 (Talk) 10:13, 25 September 2012 (UTC)
- At Wikipedia, the best place to ask copyright questions is at WP:MCQ. There are some knowledgable people (not lawyers, they're advice is useful but meaningless in a legal context) who frequent that board. --Jayron32 19:49, 24 September 2012 (UTC)
- Where should I place it, then? --Slgrandson (How's my egg-throwing coleslaw?) 18:33, 24 September 2012 (UTC)
- This isn't legal advice, but international copyright law (and the question of which country's laws matters, when the dates don't like up) is handled by the Berne Convention for the Protection of Literary and Artistic Works. It can be slightly complicated, but generally most countries follow the rule of the shorter term, which means that if it is public domain in the country of origin, then it is public domain everywhere else. I provide this to you not as a means of telling you how to run your business, but just to give you the terminology you need to ask other professionals educated questions about this issue. I don't know the specifics of Spain at all. --Mr.98 (talk) 02:00, 25 September 2012 (UTC)
- Actually, it is legal advice because it concerns the questioner's actual rights and obligations - it is not an academic legal question in the abstract. The questioner needs to get some legal advice - presuambly Amazon has standing advice for Spain that can help? --PalaceGuard008 (Talk) 10:13, 25 September 2012 (UTC)
- What I meant was, "my response is not legal advice," not, "this is not a request for legal advice." What the OP needs to do is find someone who can just answer the question, "is a work published in the United States that is now in the public domain because of copyright expiration still under the copyright term in Spain?" I suspect the answer is no, for the reasons given above, but I am not a lawyer, and I should not be trusted with this. But an intellectual property lawyer can probably answer this question in about five seconds. --Mr.98 (talk) 14:17, 25 September 2012 (UTC)
- Actually, it is legal advice because it concerns the questioner's actual rights and obligations - it is not an academic legal question in the abstract. The questioner needs to get some legal advice - presuambly Amazon has standing advice for Spain that can help? --PalaceGuard008 (Talk) 10:13, 25 September 2012 (UTC)
Laughter
Why is laughter so contagious and how is it spread? Ankh.Morpork 18:17, 24 September 2012 (UTC)
- Contagion is a result of systems that are built into our brains, but how it works is not very well understood. (I have linked to a disambig page because we have several related articles, although all of them suck.) There is a possibility that it is related to so-called mirror neurons, which are brain cells that are activated both when a person performs an action and when the person observes somebody else perform that action. Looie496 (talk) 18:25, 24 September 2012 (UTC)
- It probably has similar causes as the contagiousness of yawns. Yawn#Contagiousness has some information there. --Jayron32 18:38, 24 September 2012 (UTC)
- I think that yawning is a lot more contagious than laughing -- in some contexts, one feels no urge to laugh when other people are laughing, while it's easy to yawn without thinking after someone else (even of another species) yawns... AnonMoos (talk) 23:46, 24 September 2012 (UTC)
- While studies have shown that people who are in the same room as yawning people are more likely to yawn, or even reading a book or watching a movie about yawning will make you yawn, it's not clear what is the reason for yawning. --Activism1234 23:47, 24 September 2012 (UTC)
- As far as an evolutionary cause, laughing along with others is an important part of social bonding, and having a strong social bond with others has historically been important to both survival and procreation, and thus passing on your genes (less so these days). StuRat (talk) 19:49, 24 September 2012 (UTC)
- I remember reading somewhere about a behaviorist (I think B.F Skinner) that instructed nobody to smile at his infant to assess whether laughter and smiling was learned or innate. Has there been any studies on this? Ankh.Morpork 23:52, 24 September 2012 (UTC)
- I dunno, but if he was anything like Watson, Mariette Hartley's grandfather, he must have been miserable to be around. That sort of thing nowadays would be considered child abuse. As to learned or innate, have you noticed that laughter sounds the same no matter the language? ←Baseball Bugs What's up, Doc? carrots→ 00:32, 25 September 2012 (UTC)
- Not according to The Simpsons, where Nelson says "Ha Ha !" and his French equivalent says "Ho Ho !" StuRat (talk) 00:40, 25 September 2012 (UTC)
- I don't agree that it sounds the same. Many can distinguish from the chortle of a Russian and the hearty laughter of a Frenchman. --Activism1234 00:50, 25 September 2012 (UTC)
- There are different individual styles of laughter within any group of language speakers and even for the individuals. Listen to audio of a crowd laughing, especially a large group of children, without any visual or language reference, and I think you would be hard pressed to say with certainty what their language or ethnic group is. ←Baseball Bugs What's up, Doc? carrots→ 01:29, 25 September 2012 (UTC)
September 25
Thucydides on religion
According to Thucydides#The_History_of_the_Peloponnesian_War, Peter Green claimed "Religion was women's business, and mostly nonsense anyway, so that could be discarded too." Did Thucydides ever claim this in his history of the Peloponnesian War, or is this Green making up feminist bullshit? I know that Thucydides was an atheistic and objective historian, at least compared to Herodotus, but is there any evidence that Thucydides (or any other Athenian) considered religion to be women's business, or that he discarded it for that reason? Was it even true that Athenian women were more religious than men? --140.180.242.9 (talk) 00:56, 25 September 2012 (UTC)
- I'm not sure what such a statement would be referring to. There were certain private women-only celebrations such as the Thesmophoria, but the public ceremonial rituals seem to have been dominated by men... AnonMoos (talk) 03:05, 25 September 2012 (UTC)
- "Religion" and "women" aren't in the index of my library's copy of The History of the Peloponnesian War, nor are they mentioned on Thucydides's Wikiquote page. Green's source for this claim is unclear, so I'd take it with a grain of salt. --BDD (talk) 20:53, 25 September 2012 (UTC)
- "Feminist bullshit"? sounds like the opposite of feminist to me. Green is clearly editorialising a little here but he adding up some of the significant facts about the work to depict a plausible view of Thucydides' opinions. In the hundreds of pages of the War and the many years depicted, he hardly mentions either women or religion. As you say this comes across as atheistic compared to Herodotus. He tends to avoid superstitions about things like fate, seen in similar works, and finds practical reasons for events. One of the very few women mentioned more than a name is Chrysis (priestess) who ran away after being a klutz. This is an odd little gossipy diversion from his usual rigorous style and it is speculated it had some extra meaning, perhaps an attack on a fellow historian or a general laugh at foolish women and religion. This is probably what Green is basing his characterisation of Thucydides on as there is very little other detail of his life and views other than his book.
- As to women's actual role in Athens and religion, we don't know very clearly and indeed Pericles' Funeral Oration at least partly edited if not wholly written by Thucydides says "the greatest glory of a woman is to be least talked about by men". It is know they were mostly confined to their houses and one of the few outlets they had in the public sphere was in religious ceremonies so they probably did have a greater involvement than men. That is not to say men did not involve themselves in religion and records of purchased priesthood ranks show buying a male priest's job was always more. Girls and Women in Classical Greek Religion by Matthew Dillion is the work to read on this subject. meltBanana 00:02, 26 September 2012 (UTC)
- From The World of Athens by JACT, p.86: "Since there were female and male gods, there were also male and female priests." Obviously they are implying the genders were paired to those of the gods for worship. I don't know if it was universal throughout Greek religion, however. Just thought I'd share the quote, and maybe someone can add to it. IBE (talk) 02:41, 26 September 2012 (UTC)
- It's commonly observed that women are more religious than men across religions, nationalities and times (the reasons for this are disputed, and the link is chosen merely as an illustration of the observation) - although the hierarchy of organised religions are usually male or male-dominated, the rank-and-file believers and worshippers skew female. Perhaps Green is simply assuming that's true of Greece at the time of the Peloponnesian War. --Nicknack009 (talk) 12:11, 26 September 2012 (UTC)
Question on history of aviation
In the history of aviation, when was enclosed cockpits begin to be implemented for heavier-than-air airplanes? K61824 (talk) 02:32, 25 September 2012 (UTC)
- According to cockpit, it was the Avro Type F in 1912, though open cockpit aircraft were used alongside closed cockpit models for some time. --Jayron32 02:44, 25 September 2012 (UTC)
- The Sikorsky Ilya Muromets of 1913 was another early aircraft with an enclosed cockpit and the first to have an on-board toilet. Military aircraft kept the open cockpit much longer; the Fairey Swordfish outlived its closed cockpit successor, the Fairey Albacore, and was still in combat service in 1944. Alansplodge (talk) 14:31, 25 September 2012 (UTC)
A new or an old social class?
If you look at people with a huge negative net worth, who are tied to their house (+mortgage) but are formally free to go where they wanted (but just can't go anywhere); do they resemble a past social class, some medieval peasants, who were in the same situation? Is there a name for that? (that = free but tied to some place). Is any sociologist already studying them? OsmanRF34 (talk) 16:33, 25 September 2012 (UTC)
- Does wage slavery work for you: people who are legally free to move about, but whose financial situation prevents them from doing so? Also related is company town, whereby people who live in an ostensibly free society have their lives so dominated by their employers that they are functionally tied to the company. The agricultural equivalent of that is the sharecropper. There are many situations where people's freedom is restricted as though they were legally slaves, excepting that slavery is completely illegal, as the economics of their lives is so totally dictated by their employment as to prevent any social or physical mobility at all. I think I've given some examples. In some ways, the medeival system of serfdom is related, but I think in those situations there were real legal restrictions placed on serfs by the state. Is this sort of stuff what you are looking for? Fifelfoo will be along shortly to explain this using obscure Marxist jargon. --Jayron32 16:43, 25 September 2012 (UTC)
- Thanks! Fifelfoo (talk) 01:29, 26 September 2012 (UTC)
- Having a debt and not being able to do anything else until you are free of that debt? Indentured servant and Corvée might have something to do with this. Astronaut (talk) 17:38, 25 September 2012 (UTC)
- How are you tied to your house by a mortgage? If you want to go somewhere else and live there, that's fine. If you no longer want the house, that can usually be negotiated if you agree to pay a penalty. If you really can't pay the mortgage under any circumstances, bankruptcy is an option. Even if you outright refuse to pay the mortgage and ignore the lawsuit, the most that will happen is that your assets or earnings will be seized to pay the debt, which is nothing like serfdom. --140.180.242.9 (talk) 18:36, 25 September 2012 (UTC)
- Well, people are also tied to a social standing, and may be unwilling to accept the consequences for a course of action, and so may remain in a situation which they don't want to because they want the alternatives less. If your only two choices are "I can kick you in the teeth, or I can poke you in the eyes. One of these two will happen: choose now!", many people would not consider that choice a form of freedom: You're going to get hurt pretty bad either way, and the choice is a non-choice in that regard. With people like the OP is refering to, they are often faced with being stuck in a house which they don't want to leave becuase they are "underwater" and cannot possibly sell the house to cover the outstanding bill on the mortgage (the kick to the teeth), but the only way out is to declare bankruptcy and completely ruin their credit to the point that they can't buy a similarly sized house again, thus would lose the lifestyle they have become dependent on (the poke in the eye). For many people, there is no real freedom here, because there is no availible choice which allows a way "out" of a bad situation in a satisfactory manner. Legally, they have the right to sell or declare bankruptcy or move anywhere they want; but they are restricted by their economics and their social situation. I think that is what the OP is talking about here. --Jayron32 18:47, 25 September 2012 (UTC)
- (ec) Negative equity "traps" many people (listen). However, perhaps "trap" is too strong a word. You can still move if you want, but must be prepared to take a loss and possibly remain in debt to the mortgage company even though you no longer own the house. Without an asset to back up your remaining loan, the mortgage lender could then call in the loan. For this reason, many people in negative equity prefer to keep the house in the hope that the housing market will improve, but then have to turn down work if it is too far away. Astronaut (talk) 18:58, 25 September 2012 (UTC)
- Well, people are also tied to a social standing, and may be unwilling to accept the consequences for a course of action, and so may remain in a situation which they don't want to because they want the alternatives less. If your only two choices are "I can kick you in the teeth, or I can poke you in the eyes. One of these two will happen: choose now!", many people would not consider that choice a form of freedom: You're going to get hurt pretty bad either way, and the choice is a non-choice in that regard. With people like the OP is refering to, they are often faced with being stuck in a house which they don't want to leave becuase they are "underwater" and cannot possibly sell the house to cover the outstanding bill on the mortgage (the kick to the teeth), but the only way out is to declare bankruptcy and completely ruin their credit to the point that they can't buy a similarly sized house again, thus would lose the lifestyle they have become dependent on (the poke in the eye). For many people, there is no real freedom here, because there is no availible choice which allows a way "out" of a bad situation in a satisfactory manner. Legally, they have the right to sell or declare bankruptcy or move anywhere they want; but they are restricted by their economics and their social situation. I think that is what the OP is talking about here. --Jayron32 18:47, 25 September 2012 (UTC)
- the OP said :huge negative net worth.
- Yes, very much so. Since ancient Egyptian times, economic cycles have been recorded when the very wealthy have become the Nouveau poor. This cycle have been repeated into modern times. Within living memory, many Gentile families in the 1930's found them selves 'financially embarrassed' when their investments went Pear-shaped. Please Note: they did not become 'poor' but simply 'financially embarrassed' because poor people (as everybody knows) only have themselves to blame for their plight but the [Nouveau poor] also have somebody else to blame – as always. Fortunately, in this present and more enlightened age, the Government is now willing to step -in and divert taxes from the poor (who are too stupid to know what to do with their money) and bail out the more worthy who have fallen on bad times due to no fault of their own, other than a having a health appetite for greed during the good times. After all, if these oligarchs are not helped through these difficult times -that we all find ourself in- how are the going to lead us to the promised land, where their wealth (in the next economic boom) will drizzle down to us like Niagara Falls in a flood ? Or like dew on the morning grass if-it-don't-rain -like in the last 4,500 years or more. --Aspro (talk) 19:58, 25 September 2012 (UTC)
- I understand the sentiment, but that probably could have been said with less snark. We're supposed to be presenting links to references, either within Wikipedia or outside of it, and not necessarily presenting a political argument one way or another for the benefits or liabilities of any one particular economic system. And doubly so in the tone we take when presenting what we have to say. --Jayron32 20:04, 25 September 2012 (UTC)
- People have been telling me that Debt: The First 5000 Years is a very good book related to this topic. I haven't looked at it yet but I plan to do so sometime. 69.228.171.70 (talk) 21:10, 25 September 2012 (UTC)
- To pre-empt Fifelfoo on the Marxist analysis, people in negative equity don't constitute a class in their own right but are part of the working class aka proletariat. In fact they are not the most exploited fraction of that class but are what are (sloppily) called the "squeezed middle". As already pointed out, they do have some choices, probably more than those who have never got on the "housing ladder" in the first place. Itsmejudith (talk) 21:22, 25 September 2012 (UTC)
- They might have better choices than the 'always poor', but, if everything else is equal, it's still better to have $0 than to have -$500.000, which is the case here. And I'm not sure there is a "squeezed middle" in Marxist theory. I don't believe most sociologists/philosophers/economists thought at a society with so many social classes, with so many aspects as modern societies. OsmanRF34 (talk) 00:33, 26 September 2012 (UTC)
- Being $50 million in debt might not be that bad a deal. It means you have some characteristic that got your hands onto $50 million so you could lose it in the first place, so maybe you can do the same thing again. That approach seems to work for any number of Wall Street guys and the like. By comparison, if you have $0, you are just a sad sack. 69.228.171.70 (talk) 00:47, 26 September 2012 (UTC)
- They might have better choices than the 'always poor', but, if everything else is equal, it's still better to have $0 than to have -$500.000, which is the case here. And I'm not sure there is a "squeezed middle" in Marxist theory. I don't believe most sociologists/philosophers/economists thought at a society with so many social classes, with so many aspects as modern societies. OsmanRF34 (talk) 00:33, 26 September 2012 (UTC)
- When is Fifelfoo with his Marxist explanation coming? OsmanRF34 (talk) 00:38, 26 September 2012 (UTC)
- Right about now, though I will note the value of Itsmejudith's contribution from a Marxist perspective. Negative net worth is a phenomena that only exists in capitalism. In prior societies people in debt could merely kill the people they owed money to. Sovereign defaults have been rife, where the monarch was capable of threatening or executing those who they were in debt to. In later years merely threatening a partial default was sufficient, rather than murdering them. Lesser nobles also conducted this—debt payments were as regular as the threat of witholding of future credit made them. So negative net worth should only be discussed in relation to "advanced" societies of the last 400 years or so. (Societies prior to feudalism also had concepts of a similar nature, especially Roman commercial law, see attempts to enslave citizens for which the sharecropper metaphor is relevant.) Let us consider the classes in capitalism, and who could be in negative net worth (hint: all of them.)
- When the bourgeoisie is in negative net worth it is in a relationship with other capitalists as finance capital they become bankrupt and any capital goods or commodities they possess are seized and distributed amongst their creditors. This is relatively standard and you see it all the time, including for corporate persons. (the unpaid wages come way down the list of creditors whose debts get paid)
- When the petits-bourgeois is in negative net worth, often it is in a power relationship with capital proper. Often it is more in the interests of capitalists to allow small business to operate, see share cropping or Brecht's Three penny novel. Often chain store franchisees feel this, and it reminds me strongly of patronage and the patron systems. Again, ancient Rome's commercial law is relevant, as is the dispossession of free peasants in feudalism. One bad year.
- We can consider the modern peasantry as simply the petit-bourgeois or the lumpen proletariat, Emile Zola's Earth is a useful discussion of this pain.
- The proletariat regularly goes through debt cycles. Some are basic and related to day to day expenses—the lumpenproletariat often experiences this kind of negative net worth, got any money for Heroin?. Often the proles can skip the rent, stuff the small store creditors, and shift town. However, as Itsmejudith points out, some workers have sunk costs in immobile commodities, like housing mortgages, "skills" that are only in demand in particular locations, a culture or society they don't wish to leave. The bondage here in relation to net worth seems new to me. Unlike sharecroppers, the house isn't productive capital (never mind the Autonomists who think of the wage labouring household as a producer of labour power). In some ways the fixity is similar to the problem of medieval free peasants. The other kinds of bondage seem to be preferences or bonded labour. I prefer to live within 30 minutes bicycle commute of where I work. The number and variety of jobs open to me in 2012 is far fewer than the number of jobs and variety that would be open to me (anachronistic as hell) in 1950s Australia.
- Finally some people see a Professional-managerial class existing in capitalism. These people are liable to negative net worth and seem to behave more like the richer proletarians.
- So the answer is: negative net worth is a common feature across all classes in capitalism, there are some useful metaphors relating to previous classes—in particular enslavement of citizens in slave societies and the patronage system in agricultural societies such as feudalism. But, as Itsmejudith notes, the largest class in capitalism is the proletariat, so the proletarian experience here can often become definitive of people's understandings of what happens when negative net worth situations arise (stuff the dominant ideology hypothesis). I hope this gives you enough things to inquire into in relation to Marxist views of negative net worth and class? Fifelfoo (talk) 01:29, 26 September 2012 (UTC)
- Osman, it seems to me that you've made a basic oversight: why are you tied to living in a certain place just because you own a residence there and have a big mortgage on it? I've known plenty of people who resolved this kind of situation by renting the property to someone else while they were gone. Nyttend (talk) 02:47, 26 September 2012 (UTC)
- What if rent doesn't cover the mortgage? You still lose money all the time, and if you're already deep in debt, you will simply go deeper in debt... --Jayron32 02:49, 26 September 2012 (UTC)
- I was talking about one of those properties of the bubble, which lost maybe 50% and still no one wants them. Moving out is not possible, since you'll still have to pay the mortgage. Declaring bankrupcy is sometimes not possible. Just imagine you have a small business which reports $1000/months, you'll lose it and be in a even worse situation. OsmanRF34 (talk) 03:45, 26 September 2012 (UTC)
- Where I live, rent is twice mortgage, but a mortgage should be paid off in 30 years. So compare starting at 20, living to 80, saving for ten years for a mortgage, then paying it off, then not having housing costs for access to housing at: 10x + 30*2x + 20*0x = 70x versus renting for life as 60x but having to move every 2 years because landlords (where I live) are nasty arseholes. Many people say, that at the end of their live they will possess about 30x worth of assets whereas a renter for life will possess 0x worth of assets. YMMV, realestate agents, landlords and mortgage banks are all evil bastards. Fifelfoo (talk) 03:46, 26 September 2012 (UTC)
- Speculators and bubble properties that can't be sold for the purchase price didn't occur to me; I thought you were asking about people who buy bigger-than-necessary houses that they can't really afford, but that aren't so massive that renting is abnormally hard. Nyttend (talk) 03:52, 26 September 2012 (UTC)
- In some places, like Spain, you cannot just give the appartment back to the bank, you'll still owe the mortgage after it is sold at a public auction after a foreclosure. And you cannot file for bankrupcy, since that doesn't cover mortgage debt. Indeed, there's no way of getting rid of a mortgage without paying it, or finding another person that buys the appartment from you and pays the whole mortgage (which is increasingly difficult with prices falling and all). That's the more dramatic scenario of debt slavery, you are stuck with your appartment, cannot move to another city even if you find a job. 80.31.74.108 (talk) 00:15, 27 September 2012 (UTC)
- Speculators and bubble properties that can't be sold for the purchase price didn't occur to me; I thought you were asking about people who buy bigger-than-necessary houses that they can't really afford, but that aren't so massive that renting is abnormally hard. Nyttend (talk) 03:52, 26 September 2012 (UTC)
- What if rent doesn't cover the mortgage? You still lose money all the time, and if you're already deep in debt, you will simply go deeper in debt... --Jayron32 02:49, 26 September 2012 (UTC)
- Osman, it seems to me that you've made a basic oversight: why are you tied to living in a certain place just because you own a residence there and have a big mortgage on it? I've known plenty of people who resolved this kind of situation by renting the property to someone else while they were gone. Nyttend (talk) 02:47, 26 September 2012 (UTC)
Embassy premises sharing
Canada recently announced that it will share some embassies with the United Kingdom; what other countries do that? I'm pretty sure the diplomatic mission of the U.S in Cuba is housed in the Swiss embassy. Eisenikov (talk) 20:23, 25 September 2012 (UTC)
- The Cuba example is something slightly different; the Swiss embassy acting for the US because the US has no relations with Cuba. --Tagishsimon (talk) 20:26, 25 September 2012 (UTC)
- Diplomatic mission#Non-diplomatic offices gives an overview of some situations like that. Current Google searches for "embassy sharing" are clogged with news stories about Canada, but I also found this news item from 2009 about Central European countries sharing embassies. Again, it seems to be about cost-cutting. --BDD (talk) 20:44, 25 September 2012 (UTC)
- According to [12], NZ colocates with the UK in Kabul. Nil Einne (talk) 00:04, 26 September 2012 (UTC)
- Diplomatic mission#Non-diplomatic offices gives an overview of some situations like that. Current Google searches for "embassy sharing" are clogged with news stories about Canada, but I also found this news item from 2009 about Central European countries sharing embassies. Again, it seems to be about cost-cutting. --BDD (talk) 20:44, 25 September 2012 (UTC)
- We need to be careful with terminology. The embassy is the people, rather than the building. So, the situation with the US in Cuba is actually the Swiss embassy providing consular services for Americans. That is different to, for example, the Embassy of Ecuador, London (where Assange is), which shares a building with another embassy (I can't remember which country and Google is letting me down...), but the two embassies (ie. the people) are completely separate. --Tango (talk) 00:20, 26 September 2012 (UTC)
- So what happens when two countries that share an embassy, then one of them falls out of favour with the host country (or fall out of favour with each other)? Astronaut (talk) 02:51, 26 September 2012 (UTC)
- The embassy (people) go home. The embassy (building) is (partially) empty and/or gets a new tenant. --Stephan Schulz (talk) 06:33, 26 September 2012 (UTC)
- You need a lot more than "falling out of favour" before you break off diplomatic relations. Even if you withdraw your ambassador, there will still be more junior members of the mission there. To completely withdraw an embassy you need to be practically on the verge of war (eg. the US has formal relations with pretty much everywhere except Iran, Cuba and North Korea, I think, and those are all countries it has been at war (or had proxy wars) with in the last few decades - it can take a long time for relations to fully recover). --Tango (talk) 11:32, 26 September 2012 (UTC)
- The other embassy in 3 Hans Crescent is the Colombian one. 130.88.99.231 (talk) 18:11, 26 September 2012 (UTC)
- So what happens when two countries that share an embassy, then one of them falls out of favour with the host country (or fall out of favour with each other)? Astronaut (talk) 02:51, 26 September 2012 (UTC)
- Here's an example from Warsaw: the embassies of Cuba, Ecuador, India, Mongolia, Uruguay, and Venezuela occupy various floors of this drab office tower at Rejtana 15, which they share with private businesses on other floors.
- For cases like the one mentioned above of Switzerland representing U.S. interests in Cuba, see protecting power. It's a somewhat amusing name, as it's often smaller countries that act as "protecting powers" of powerful empires. My country, Poland, for example, used to act as the protecting power for the United States in Iraq and, more recently, in Syria. — Kpalion(talk) 07:13, 26 September 2012 (UTC)
- A couple of comments. the common name for an embassy building is a chancery. There is a difference between various Embassies renting out premises in a single office building (a common thing, as in the Warsaw example above, which does not imply any particular cooperation between the various tenants), and two countries operating out of a shared chancery (much more rare). It is relatively common for one country to host a diplomat from another country in its chancery, but this is more akin to the first case (the Bordurian diplomat is "renting" an office in the Syldavian Embassy to Absurdistan, for example). A shared Embassy would be more along the line of two distinct embassies using a single chancery; you would have two Ambassadors and discrete diplomatic staff, but some shared services (such as drivers, administrative assistants, etc). I know that Germany and the UK used to share an Embassy along those lines in Kazakhstan when that country first became independent, but they may well have moved to separate premises in the last two decades. By the way, it's not clear that the Canadian/UK proposal is for anything that drastic; it looks to be more along the line of a British diplomat operating out of the Canadian chancery in some country where the UK does not have an Embassy, and vice-versa. --Xuxl (talk) 10:42, 26 September 2012 (UTC)
- I don't know if this is something you are interested in, but under the Maastricht treaty, EU members' embassies are required to give consular assistance to citizens of other EU members if their own government has no diplomatic presence in the country (see European citizenship). The Commonwealth of Nations has a similar agreement - there are probably other examples of this. 130.88.99.231 (talk) 18:20, 26 September 2012 (UTC)
- To link to the previous comment, I have heard that people from the UK can seek help at both Australian and Canadian embasses (e.g. if they lose their passports), if, for whatever reason they can't get to the UK embassy. Similarly, Canadians and Australians can go to the UK embassy. I guess this has to do with the mentioned agreement amongst the Commonwealth states.
- As for embassies sharing buildings, I immediately thought of the Nordic Embassies in Berlin. V85 (talk) 19:32, 26 September 2012 (UTC)
- Also, the Embassy of Iceland in London is located in the same building as the Danish Embassy. In the picture, you can see the main entrance on the front of the building being that of the Danish Embassy, while the Icelandic embassy is on the side nearest the photographer.
- However, as Xuxl points out, the similar location of these embassies, might not imply any great cooperation between the states sharing a location. Given that a lot of what embassies deal with is classified information, embassies might not be too keen on foreign diplomats snooping around the office and 'accidentally' coming across state secrets. I don't know exactly how the UK and Canada are planning to 'share' embassies, but from what I have read just now, it seems to be a case of either country 'hosting' the other country's diplmats in their embassies in Haiti and Burma. If there is further integration, I would guess it would be something akin to a co-location, i.e. the embassies are in the same place, but operate independently of each other. V85 (talk) 19:55, 26 September 2012 (UTC)
- Without commenting on the prevalence of full embassies (missions to the host country's capital) sharing a single building or space, the following example seems instructive here: List of diplomatic missions in Boston. Sorting by address will make clear that various nations' consulates often occupy the same building (though obviously not the same particular office space). To whatever extent a city has many diplomatic missions, it may make sense to house many of them in a diplomatic quarter or neighborhood. Sharing some buildings is simply the next logical step. It was a wise decision to separate Greece and Turkey by several blocks and the entire Public Garden, with Cyprus across the river entirely. ☯.ZenSwashbuckler.☠ 18:36, 28 September 2012 (UTC)
When Iraq and Syria didn't have diplomatic relations, there was a period when there was a Iraqi representation office inside the Algerian embassy in Damascus. --Soman (talk) 17:41, 30 September 2012 (UTC)
September 26
Muslim dominated neighbourhoods in Europe
I read a book called Apart: Alienated and engaged Muslims in the West and the author conducted his research in two European neighbourhoods: U.K.'s London's East End (Most of them are Bangladeshis) and Spain's Madrid's Lavapies (most of them are Moroccans). Is there other neighbourhoods of Europe that are Muslim-dominated? — Preceding unsigned comment added by 174.89.42.135 (talk) 00:36, 26 September 2012 (UTC)
- Islam in France indicates that the largest concentrations of Muslims in France may be in the département of Seine-Saint-Denis, a small département outside of Paris. That article reports that the entire département has a population of 1.4 million, and also has 500,000 muslims, so undoubtedly some of the neighborhoods there will have a Muslim majority population. France doesn't really have an official or unoffical designation equivalent to "neighborhood" in most contexts (though some large cities have unofficial neighborhoods), in France the lowest subdivision is the commune, which outside of the big cities, many of which are "neighborhood sized". So, looking through the arrondissements and communes of Seine-Saint-Denis, you should be able to turn up many muslim-majority areas. --Jayron32 02:40, 26 September 2012 (UTC)
- [edit conflict] Surely the majority of neighborhoods in Albania and in Bosnia and Herzegovina are Muslim-majority, since those countries are. Otherwise, I'd suggest that you read Islam in Europe and country-specific articles linked at {{Islam in Europe}}. Nyttend (talk) 02:43, 26 September 2012 (UTC)
- It's probably cheating, but the cities of Ceuta and Melilla in Spain are clearly Muslim majority; though they are part of Spain they are also on the African continent, so it's not really "Europe" I suppose. --Jayron32 02:44, 26 September 2012 (UTC)
- It's not Europe at all. Spain is one of those countries spread over more than one continent. Most people aren't aware of that. See List of transcontinental countries for the others, which include Italy, France and the United States in addition to the well-known cases like Russia and Turkey. -- ♬ Jack of Oz ♬ [your turn] 03:21, 26 September 2012 (UTC)
- Our article on Bosnia and Herzegovina says that only about 45% of the population are Muslims, and many areas of the country are dominated by Christians, such as, I imagine, most of the Republika Srpska. 130.88.99.231 (talk) 12:59, 27 September 2012 (UTC)
- It's probably cheating, but the cities of Ceuta and Melilla in Spain are clearly Muslim majority; though they are part of Spain they are also on the African continent, so it's not really "Europe" I suppose. --Jayron32 02:44, 26 September 2012 (UTC)
- [edit conflict] Surely the majority of neighborhoods in Albania and in Bosnia and Herzegovina are Muslim-majority, since those countries are. Otherwise, I'd suggest that you read Islam in Europe and country-specific articles linked at {{Islam in Europe}}. Nyttend (talk) 02:43, 26 September 2012 (UTC)
In Britain, the information is collected in the Census and you should be able to find tables and maps. 2001 Census, for now, the results for the 2011 Census will appear later this year. Parts of Bradford, that is well known. Itsmejudith (talk) 06:57, 26 September 2012 (UTC)
- For England, you could start with the information at Islam in England#Demography and ethnic background. Ghmyrtle (talk) 08:41, 26 September 2012 (UTC)
- Parts of Groruddalen have 'immigrant populations' of close to 90%. Similarly, Grønland has a high percentage of immigrants (though not a majority, though that might change from neighbourhood to neighbourhood). There have been cases of Muslims exerting social control here, primarily on other Muslims, but also on non-Muslim Norwegians, such as the gay couple that was attacked and told that 'Grønland is a Muslim neighbourhood', where no expression of homosexuality is accepted.
- In Sweden, Malmö's population is 40% immigrant, in the neighbourhood Rosengård, close to 90% of the population is immigrant, most of them coming from Muslim countries. V85 (talk) 19:21, 26 September 2012 (UTC)
Lockouts in general
Regarding the concept of a lockout (industry), I'm somewhat confused by the following text:
A lockout is generally used to enforce terms of employment upon a group of employees during a dispute. A lockout can act to force unionized workers to accept changed conditions such as lower wages. If the union is asking for higher wages, or better benefits, an employer may use the threat of a lockout or an actual lockout to convince the union to back down.
If I'm the employer, and I'm trying to get my employees to accept lower wages, I can understand why I'd use a lockout; "you're not allowed to work until you accept lower wages" is an effective tool if the employees submit. But why would I use it when the workers want higher wages? Unless I submit to their demands, or unless they persuade a government to raise minimum wages, they're not going to get higher wages when working for me. Why would I refuse to let them work for me at a certain wage when I'm trying to convince them to work for me at that exact wage? Further explanation in this section (as well as sourcing; there are no citations) would be appreciated. Nyttend (talk) 02:36, 26 September 2012 (UTC)
- Why not. The employees may be asking for better wages during contract negotiations; management could use the threat of lockout to force them to lower their demands. --Jayron32?
- Still confused. Wouldn't a lockout hurt both parties in the short term? And wouldn't "you're going to work on my terms if you want to work" get the point across without hurting me as much? The worst that the workers can do (as long as they obey the law) is to go on strike, and I'm left with a situation marginally better than a lockout. Nyttend (talk) 02:51, 26 September 2012 (UTC)
- Whether it is a lockout or a strike is probably largely dependent on which party thinks the other party needs them more. If labor thinks that management can't hold out longer then them, they srrike. If management thinks they can manage without the union, they lock them out. Labor actions like this aren't concerned with the short term pain. It's all a game of "chicken": whichever side can take more "pain" without crying "uncle" wins. --Jayron32 03:34, 26 September 2012 (UTC)
- They balance the short term losses with potential long-term savings. A lock-out may be necessary if dissatisfied union workers might sabotage equipment or perform a slow-down (where employees are still paid, but don't get their work done). And, seeing the gates locked also has a psychological effect, making employees think about the possibility they could lose their jobs permanently (if the factory hires replacement works, moves, or just shuts down). StuRat (talk) 03:41, 26 September 2012 (UTC)
- Employers normally have a greater potential to accumulate capital, either by delaying realisation (stockpiling output), by scheduling preventative maintenance using an alternate workforce, by using "staff" employees to operate machinery while the day wage employees are locked out, or by hiring scabs—than workers have the capacity to save wages necessary for purchasing the useful things of life. My belly empties before my bosses' coffers do, which is why a lockout can break union solidarity. As Jayron32 notes, some industrial disputes are chosen by both workers and their boss and it is largely a technicality as to whether it is a strike or a lock-out. Employers usually face much laxer laws regarding illegal lock-outs than employees face for illegal strikes. Businesses live on sales, sales depend on stockpiles, and stockpiles depend on production. One can halt production and as long as one has a sufficient stockpile one can stuff the workers. Fifelfoo (talk) 03:52, 26 September 2012 (UTC)
- Some employers may also find it infeasible to operate when the union has a strike mandate but has not yet called a walkout - for instance a steel foundry or airline needs a stable operating schedule to function, so it is better for management to initiate a controlled shutdown rather than hope the workers keep showing up for the lower pay rate. Franamax (talk) 04:25, 26 September 2012 (UTC)
- Similarly, unions can call for a partial strike that would still disable most of a factory (e.g. only the people who put wheels on cars in a car factory - very soon, the unfinished cars on the assembly line will block the whole line). The employer may want to avoid paying the non-striking workers, especially if they are part of the same union, as a way of depleting union funds and putting pressure on the union. --Stephan Schulz (talk) 06:41, 26 September 2012 (UTC)
- How effective such a strategy would be depends on how big the union is. In Belgium, there are three, the socialist, the christian and the small liberal union (maybe some others, but these three represent at least 95% of union members). As far as I know, the funds they have are not split by sector, so depleting them would be hard to do. Ssscienccce (talk) 09:37, 26 September 2012 (UTC)
- Similarly, unions can call for a partial strike that would still disable most of a factory (e.g. only the people who put wheels on cars in a car factory - very soon, the unfinished cars on the assembly line will block the whole line). The employer may want to avoid paying the non-striking workers, especially if they are part of the same union, as a way of depleting union funds and putting pressure on the union. --Stephan Schulz (talk) 06:41, 26 September 2012 (UTC)
UK wealth tax proposal
Regarding "Lib Dem conference: Clegg promises to push for wealth tax," 23 September 2012, BBC and "A 20% wealth tax on the mega rich would raise up to £800bn," 25 September 2012, New Statesman, would someone with insight into the workings of the UK coalition government please say whether this is notable noteworthy enough to include as a proposal in Wealth tax yet? —Cupco 02:36, 26 September 2012 (UTC)
- The reference desk is NOT the venue in which to discuss issues of notability or other matters properly discussed on an article's talk page, on an admin board or workshop, or in an RfC. If you are looking for articles, sources, or references we can help you. Otherwise, please discuss this at the article and see Wikipedia:Notability. μηδείς (talk) 03:29, 26 September 2012 (UTC)
- I've raised the question at the article's talk page and don't expect to hear much back because it doesn't get much traffic. What I'm really interested in is the prospects for the proposal by someone who is familiar with the practicalities of the UK coalition government. I.e., will we ever hear of it again or will it be making the news. As a technical matter, I believe I am asking about noteworthiness rather than notability in the sense those words are used in Wikipedia, and if there is some rule which excludes questions regarding noteworthiness, please let me know about it. —Cupco 04:40, 26 September 2012 (UTC)
- It's a question about editing, not a search for info, so outside the scope of this desk. Try the Neutral Point of View Noticeboard, WP:NPOVN. Itsmejudith (talk) 06:47, 26 September 2012 (UTC)
- Okay, but I just thought there would be more people familiar with the ins and outs of the UK coalition processes here. I'm sure that the answers to editing questions can depend on the answers to subject matter questions as in this case. —Cupco 07:10, 26 September 2012 (UTC)
- If you don't receive adequate replies at "Talk:Wealth tax#Nick Clegg/UK" (and, personally, I would wait at least a week or so), you can always bring it up at "Wikipedia talk:UK Wikipedians' notice board". Gabbe (talk) 08:34, 26 September 2012 (UTC)
- Of course, you don't need anyone's approval beforehand to edit an article. As long as you are prepared to discuss the issue should anyone object. Gabbe (talk) 08:35, 26 September 2012 (UTC)
- Okay, but I just thought there would be more people familiar with the ins and outs of the UK coalition processes here. I'm sure that the answers to editing questions can depend on the answers to subject matter questions as in this case. —Cupco 07:10, 26 September 2012 (UTC)
- It's a question about editing, not a search for info, so outside the scope of this desk. Try the Neutral Point of View Noticeboard, WP:NPOVN. Itsmejudith (talk) 06:47, 26 September 2012 (UTC)
- I've raised the question at the article's talk page and don't expect to hear much back because it doesn't get much traffic. What I'm really interested in is the prospects for the proposal by someone who is familiar with the practicalities of the UK coalition government. I.e., will we ever hear of it again or will it be making the news. As a technical matter, I believe I am asking about noteworthiness rather than notability in the sense those words are used in Wikipedia, and if there is some rule which excludes questions regarding noteworthiness, please let me know about it. —Cupco 04:40, 26 September 2012 (UTC)
- The answer's no, as I've said on the article talk page. Ghmyrtle (talk) 09:51, 26 September 2012 (UTC)
- Thank you. —Cupco 10:18, 26 September 2012 (UTC)
Relative frequency of male prostitution
What proportion of prostitutes (preferably globally, but otherwise at some national level) are male? --149.135.146.2 (talk) 09:28, 26 September 2012 (UTC)
- I can't find this online. I recommend asking for the full text of http://www.tandfonline.com/doi/abs/10.1300/J082v53n01_02 at WP:RX. —Cupco 10:17, 26 September 2012 (UTC)
- Do you mean male-to-female transsexual prostitutes, or male prostitutes? Astronaut (talk) 18:32, 26 September 2012 (UTC)
- Why would you assume or suspect the former? AlexTiefling (talk) 19:28, 26 September 2012 (UTC)
- Indeed. A more apt split would be males providing sexual services for women, vs males providing sexual services for other males. -- Jack of Oz (Talk) 20:10, 26 September 2012 (UTC)
- Why would you assume or suspect the former? AlexTiefling (talk) 19:28, 26 September 2012 (UTC)
- Do you mean male-to-female transsexual prostitutes, or male prostitutes? Astronaut (talk) 18:32, 26 September 2012 (UTC)
Free speech vs fair elections
On the issue of free speech vs fair elections, democratic countries can be broadly classified into two camps:
- Countries where you can run political ads anytime without restriction - Australia, Canada, US
- Countries where political ads are tightly regulated during election season
Is there a WP article, website, or book that lists the countries in each category? So far I'm just googling "election law of X" for each country so I'm wondering if there's a source that compares the various election laws, especially with regards to campaign advertisements. This question is inspired by this recent election story[13] from Brazil.A8875 (talk) 11:22, 26 September 2012 (UTC)
- There is a restriction in Australia. "Under Schedule 2 of the Broadcasting Services Act 1992, which is administered by the Australian Communications and Media Authority (ACMA), election advertising in the electronic media is subject to a 'blackout' from midnight on the Wednesday before polling day to the end of polling on the Saturday. This three-day blackout effectively provides a "cooling off" period in the lead up to polling day, during which political parties, candidates and others are no longer able to purchase time on television and radio to broadcast political advertising." - http://www.aec.gov.au/faqs/election_advertising.htm --TrogWoolley (talk) 18:28, 26 September 2012 (UTC)
- Thanks. I read both Australian_constitutional_law and Australian_Capital_Television_Pty_Ltd_v_Commonwealth and I still ended up with the wrong impression. Hence why I'm looking for an authoritative source that summarizes each country's campaign advertising laws. A8875 (talk) 00:53, 27 September 2012 (UTC)
- Australia, and its states, possess common law and relative judicial independence—so you're effectively requesting legal advice. The response would be: contact an Australian Commonwealth or State based solicitor who specialises in constitutional law or elections. Australia has relatively heavily regulated election funding rules by the way, which also acts as a limit on the freedom of the bourgeoisie to shit down the airwaves. Fifelfoo (talk) 02:25, 27 September 2012 (UTC)
- Thanks. I read both Australian_constitutional_law and Australian_Capital_Television_Pty_Ltd_v_Commonwealth and I still ended up with the wrong impression. Hence why I'm looking for an authoritative source that summarizes each country's campaign advertising laws. A8875 (talk) 00:53, 27 September 2012 (UTC)
And fwiw, your question title is somewhat misleading, or contains probably bogus assumptions. You appear to be suggesting that free speech (defined in the context of your question as the ability to run ads without restriction) is necessary for fair elections. I tend to think that there are at least a couple of objections to that: 1) so long as all players face the same restrictions, the playingfield is level and 2) if we look at the USian example of SuperPACs and corporations as people, I think any disinterested observer would come to the conclusion that despite "free speech" the playingfield has been tilted enormously to the advantage of the monied classses. --Tagishsimon (talk) 13:35, 27 September 2012 (UTC)
- That doesn't show it's not necessary - it would show it is not sufficient. However, I think a person can reasonable argue that when one corporation is granted a broadcast license or a cable franchise, while many other people are not permitted to express their views on television even if they cobble together a transmitter, that this in fact was never properly free speech in the first place. Wnt (talk) 16:33, 27 September 2012 (UTC)
how dual licensing works
This question has been removed as soliciting legal advice. We cannot advise users on legal specific legal matters including copyright law. If you wish to contest this, please discuss the matter on the talk page here. μηδείς (talk) 16:46, 26 September 2012 (UTC)
First African Woman Novelist is from Imo state
My name is Henry Agbasoga, I would like to ask this question, Why is it somebody like Flora Nwapathe first African Woman novelist is not amoung the name mentioned as accademia or Hero? — Preceding unsigned comment added by 41.202.119.190 (talk) 13:50, 26 September 2012 (UTC)
- Note to others: the relevant article is Flora Nwapa -- Finlay McWalterჷTalk 13:54, 26 September 2012 (UTC)
- I've added her to Imo State#Notable people under "Arts". With regard to why she wasn't there, it's perhaps because no one had though to add her before now, perhaps because the state didn't exist when she was born in Oguta. (She does seem to qualify among "notable people from Imo State", though, as some others in that list predate the establishment of the polity.) Deor (talk) 15:48, 26 September 2012 (UTC)
Ndlovukati
Is there a title (or a concept) traditionally employed in parts of West Africa that compares to the Ndlovukati of Swaziland? Agyen Kokobo, who lived on the Gold Coast some centuries ago, would benefit from the introduction of such a term, if it exist. Nyttend (talk) 15:20, 26 September 2012 (UTC)
- Ohemaa, literally 'female ruler'. Google: "ohemaa "queen mother"", or do you need some references?—eric 20:07, 27 September 2012 (UTC)
- Here's a good books link.—eric 20:16, 27 September 2012 (UTC)
Charitable contributions
Let's say that a person wants to contribute money online, via the web, to some charitable cause (e.g., cancer, AIDS, abused kids, abused animals, whatever). When one visits the various websites that would come up in a Google search, how would one know if the organization is "legit", before donating any money to them? I'm not referring to "big names" that are no brainers, like the American Cancer Society, the Red Cross, the Humane Society, etc. But, I am referring to "lesser known" diseases/causes and organizations that don't have such high visibility and name recognition. Any suggestions or advice? Thanks! Joseph A. Spadaro (talk) 16:08, 26 September 2012 (UTC)
- In England and Wales, the Charity Commission is the registrar and regulator of charities. All registered charities are required to submit an Annual Report and Accounts every year. You would go to their website, type in the name of the charity and then you would be able to check the status of the charity. Smaller local groups are often not required to register however, and campaigning groups such as the Campaign for Real Ale and Amnesty International can not become charities because of the campaigning nature of their work. There may be a similar regulatory framework and organisation in your area (wherever that is). --TammyMoet (talk) 16:36, 26 September 2012 (UTC)
- Thanks. Sorry if my post was unclear. I am referring to the United States. Thanks. Joseph A. Spadaro (talk) 16:45, 26 September 2012 (UTC)
- The IRS website says "you may verify an organization's tax-exempt status and eligibility to receive tax-deductible charitable contributions by asking to see an organization's IRS letter recognizing it as tax-exempt. You may also confirm an organization's status by calling the IRS (toll-free) at 1-877-829-5500." Though that doesn't necessarily imply that the charity is "legitimate" (I imagine there have been cases of charities losing this status because of fraud). Obviously, a separate problem is checking that the website is actually associated with the charity in question, and that the payment method is safe. 130.88.99.231 (talk) 18:01, 26 September 2012 (UTC)
For a number of websites that rate, reviews and evaluates charities, see Category:Charity review websites. Gabbe (talk) 21:14, 26 September 2012 (UTC)
- There's the International Committee on Fundraising Organizations (which I'm surprised we don't have an article about) which is an association of national agencies that monitor fundraising for charitable purposes. There are links to the national agencies (among others the US Better Business Bureau). Sjö (talk) 08:12, 27 September 2012 (UTC)
Relevance in Buddhism
I know that Buddhists are not indifferent, even if some accuse them to be, or pretend to be. They have preferences but know that things won't always work out as wished. You simply don't have to cling to your expectations. But what's the opinion regarding notability (not in the wiki sense)? Is the eruption of a thought extinct volcano equally notable than a rainbow over a tropical forest? All these are things that exist, that are part of reality. hey are equally expression of Earth being like it is. Ptg93 (talk) 22:37, 26 September 2012 (UTC)
- Are you asking about official dogma, or about what people actually do? Looie496 (talk) 23:41, 26 September 2012 (UTC)
- I suppose people do all sort of things, so, I want to know something about scriptures. Ptg93 (talk) 12:19, 27 September 2012 (UTC)
- Not sure I understand your question, but does Equanimity (Buddhism) help?--Shantavira|feed me 07:50, 27 September 2012 (UTC)
- It's something like that, but more in the sense that everything are things flowing, the bad and the good, the big and the small. Ptg93 (talk) 12:19, 27 September 2012 (UTC)
- You mean like Impermanence?--Shantavira|feed me 13:08, 27 September 2012 (UTC)
- Buddhism touches this topic through several concepts. Shantavira gave you two, and Buddhist terms and concepts) gives you a systematic list. OsmanRF34 (talk) 14:41, 27 September 2012 (UTC)
- Ptg93—you say:
- "Is the eruption of a thought extinct volcano equally notable than a rainbow over a tropical forest? All these are things that exist, that are part of reality. hey are equally expression of Earth being like it is."
- Can you expand on that, or state it a different way? Bus stop (talk) 14:48, 27 September 2012 (UTC)
- I heard something like everything is just a flow of nature, the volcano, the tide, a man stabbing other man, or the rain falling. It's just how things are. Ptg93 (talk) 18:56, 27 September 2012 (UTC)
- There are some variations among sects, but the most fundamental Buddhist doctrine is that all existence is suffering, and that the only escape from the wheel of suffering is to become indifferent to earthly events, whether good or bad. That's a rather loose formulation -- our article on the Four Noble Truths will give more information. Essentially the first Noble Truth is that all existence is unhappiness; the second Noble Truth is that the root cause of unhappiness is craving; the third Noble Truth is that to cease being unhappy people must cease craving; the fourth Noble Truth is that to cessation of craving can be achieved by following the Noble Eightfold Path. All these again are loose formulations; see the articles for more precise statements. Looie496 (talk) 16:28, 27 September 2012 (UTC)
- I wouldn't say you have to be indifferent, you have to be independent of them, not cling to them, not attach to them, but that's make yo indifferent. Ptg93 (talk) 18:56, 27 September 2012 (UTC)
September 27
I do not agree with what is going on here. Not only do I disagree, many people do not agree with. We would like to see a change in this topic. There is a picture of it made and puts out what is needs to be changed. Because this is a fact, and we like to see the change. You still want a perfect Wikipedia? You would just have to adapt this topic. Sincerely, C.R U., from The Netherlands.
]http://i47.tinypic.com/i53h3l.jpg — Preceding unsigned comment added by 62.194.231.122 (talk) 00:10, 27 September 2012 (UTC)
- Discuss this at Talk:Shahada. You might want to be clear about what exactly you think the problem with the article is. Assuming the problem is with the article. --Tagishsimon (talk) 00:12, 27 September 2012 (UTC)
- Classic content dispute. It has been BRD'ed with no resolution. Disputed translation, and the real problem isn't in the article but in: MOS:ISLAM#Translation which judging from Talk:Shahada and archives, is a contentious choice. C.R.U. - the best place to request changes would be in the talk page for the Manual of style for Islam-related articles. If there is a consensus for leaving that term untranslated (as some editors think that it should) then perhaps the MOS can change to reflect this, and then the Shahada article can be updated, too.--Robert Keiden (talk) 00:46, 27 September 2012 (UTC)
Possible to buy New York Times in Seattle in 1910?
I don't really know how newspapers work. I've noticed various newspapers in 1910 (on Google's news archives) will sometimes have stories printed verbatim, which I guess is how the mysterious AP "wire" works (like I said, I'm 100% ignorant on all this)—and something that's made fun of in modern times by Conan O'Brien and perhaps others. So maybe my question is irrelevant. I would like to know if someone in Seattle would could pick up a New York Times before boarding a train to read on his voyage. I'm asking because I cannot find the Seattle Times archived on Google, but I can find other papers (and the NYT isn't necessarily the best, but I'm picking it because it's well known), and I need to reference newspapers available in Seattle in 1910, but obviously not ones that I can't, literally, reference! I hope all this makes sense. – Kerαunoςcopia◁galaxies 06:48, 27 September 2012 (UTC)
- You can get The Seattle Times, provided you don't mind forking over a few bucks[14] or have a valid Seattle library card[15]. If it's free you want, how about the Seattle Star? Clarityfiend (talk) 07:56, 27 September 2012 (UTC)
- Reuters, AP and UPI are common news agencies. At that time, they would have used the telegraph to send stories around the world (earlier methods included carrier pigeons !). So, Seattle papers might have had stories identical to the New York Times. However, getting the New York Times itself in Seattle would be more difficult. You could have it mailed to you, in which case it would be delivered most of the way by train, and would be maybe a week old when it arrived. You might not think anyone would do this, but quite a few people who were homesick or wanted to keep up with local events back home seemed to have distant newspapers delivered just like this, at least until the Internet allowed for a better way. StuRat (talk) 08:20, 27 September 2012 (UTC)
- The New York Times#History says: "In 1910, the first air delivery of The New York Times to Philadelphia began." I don't know how widely such deliveries took place or how quickly other cities were included, but it looks like it was possible to get the NYT some distance from NY by at least the end of 1910. It might have taken a little longer before it got all the way to Seattle. --Tango (talk) 11:28, 27 September 2012 (UTC)
- Those of us who remember the pre-Internet era will remember newstands that sold out of town newspapers. However, of course, they would be several days old. Research in the Seattle city directory might reveal if there were such establishments. Yes, the idea would be to cater to the homesick or those who sought local political news. There was, after all, no other way to do it.--Wehwalt (talk) 12:19, 27 September 2012 (UTC)
- Wonderful! Thanks for all the replies! – Kerαunoςcopia◁galaxies 21:01, 27 September 2012 (UTC)
European Defence Agency - estimated savings
Hi all, as you may or may not know, there is a European Defence Agency (EDA), which is an agency of the European_Union. The EDA does work on joint procurement, coordinated research & development etc. It is, I think, an uncontroversial claim that the EDA has allowed savings on military spending to be made amongst EU Member States and indeed such savings are an explicit part of EDA's aims. However, it is very difficult to find any kind of ballpark figures of how large these savings have been or are estimated to be. Marsh and Rees provide a figure of €6bn per year, but don't really explain that calculation (The European Union in the Security of Europe, 2011, p. 44). I seam unable to find much else...does anyone have any other clues? Thanks --European Snowleopard (talk) 13:17, 27 September 2012 (UTC)
- Actually, Google Scholar finally started providing me with at least useful percentages... But if anyone finds backup or opposition to the 6 billion figure above, I'm still grateful! --European Snowleopard (talk) 13:27, 27 September 2012 (UTC)
@ recipient
(Unsure if that belongs here, or Linguistics, or Computing, or even Entertainment. It's touching four topic areas.) Does anybody know where the shortcut "@ recipient" evolved from? The usage like if there are several answers, and, say, the 4th user is replying not only to me but to one of the other users participating. They could then write something like "@Ouch:" before replying to me and "@StuRat:" before replying to StuRat. I found an example here, in Marketdiamond's reply to two replies.
For one part, I'd guess that it's the electronic equivalent of "To recipient". Is it connected to the stupid trend in the media to introduce @ as a symbol of the internet, even if someone is referring to the WWW rather than mail? - ¡Ouch! (hurt me / more pain) 14:58, 27 September 2012 (UTC)
- I hadn't come across it before Twitter. Your Twitter user name is preceded by an @ symbol. From there it's easy to see how people use it to indicate they're answering someone directly. --TammyMoet (talk) 15:04, 27 September 2012 (UTC)
- How about this: Origin of the @reply – Digging through twitter's history. That's a description of how it came about on Twitter, and mentions that ""@[name]" is a common referencing protocol on Teh Internets, and has nothing to do with Twitter." There's a follow-up post from Garrett Murray, one of the pioneers: The Real History of the @reply on Twitter. He says "I got the idea from seeing people do it over at Flickr, where it had been happening for more than a year."
- We have an article, Mention (blogging), which gives the Twitter history, but doesn't elaborate on anything previous. - Cucumber Mike (talk) 16:51, 27 September 2012 (UTC)
- Aha! Here we go. From a comment on the first article I posted: "Both the @ and the : came from conventions already established on IRC, where you would direct a public response to a specific person using @username:. In IRC clients like Colloquy, you can type the first few letters of a username, then hit Tab and the software fills in the rest of the name for you, appending a colon automatically.
- This convention migrated to Twitter, but without the autocomplete and the automatic addition of the colon, that bit of punctuation got dropped as it didn’t really help and sometimes you needed every character you could get." - Cucumber Mike (talk) 16:55, 27 September 2012 (UTC)
- Since nobody has yet mentioned it, let me give a pointer to our at sign article, which contains most of the information here and more. Looie496 (talk) 17:31, 27 September 2012 (UTC)
Thanks all. I'm still puzzled how I could miss it at the At page. It was in there, linked as @, and that's a redirect to At sign, the page mentioned by Looie. Oops.
The emythology of "@ recipient" is given as "Attention" shortened to "At" (I've read the same about the AT commands of the Hayes standard - long ago) and written as "@" to make it stand out like a capital letter. I always thought it was an acronym, maybe "Applies To"... - ¡Ouch! (hurt me / more pain) 07:59, 28 September 2012 (UTC)
- Not sure how relevant this is in terms of historical accuracy, but I found it fun nevertheless, and now I see this topic, so I thought I'd throw it out there. While re-reading the 1995 novel (so let's say it was written throughout 1994), Relic, there are a few intranet communiqués between museum staff, et al., which are basically emails addressed to personnel with (examples) "margo green@biotech@stf" and "roger thrumcap@admin@systems" and "george moriarty@exhib@stf". I was wondering if this was accurate for the time for inter-department computer messages. – Kerαunoςcopia◁galaxies 08:10, 28 September 2012 (UTC)
What exactly does pragmatism states of logic?
In a more intricate explanation, what does pragmatism or "practical philosophy" states about logic? — Preceding unsigned comment added by 112.205.88.247 (talk) 15:24, 27 September 2012 (UTC)
- Have you read through Pragmatism#Logic? --Jayron32 15:25, 27 September 2012 (UTC)
Infanta Mariana Francisca of Portugal and Infanta Maria Doroteia of Portugal
In Joseph I of Portugal it mentions the potential suitors of his daughters Infanta Mariana Francisca of Portugal and Infanta Maria Doroteia of Portugal, but Mariana Francisca's article mentions nothing about her betrothal or marriage proposal to Louis, Dauphin of France. While her sister Maria Doroteia's article mentions that she was potential bride for [[Louis, Dauphin of France (1729–1765) |Louis, Dauphin of France]] and nothing about Louis Philippe II, Duke of Orléans, which contradicts what is said in Joseph's article. My question is who are the actual matches and marriage proposals for these two sisters?--The Emperor's New Spy (talk) 16:46, 27 September 2012 (UTC)
- What do the sources say? --Jayron32 17:10, 27 September 2012 (UTC)
- That would be what I want to know.--The Emperor's New Spy (talk) 01:54, 28 September 2012 (UTC)
- Is the Wikipedia article unreferenced on these facts? --Jayron32 03:04, 28 September 2012 (UTC)
- No references at all (in Joseph I anyway). Zoonoses (talk) 04:48, 28 September 2012 (UTC)
- Most of the articles on Portuguese royals lack any citations or references.--The Emperor's New Spy (talk) 08:53, 28 September 2012 (UTC)
- I added some references to the articles on Joseph and also Maria I but can’t seem to find any sources on Maria’s younger sisters that aren’t in Portuguese or Spanish (so says Google translate). Is there a Portugal project or ref desk where you could ask, maybe? Taknaran (talk) 15:46, 28 September 2012 (UTC)
- Most of the articles on Portuguese royals lack any citations or references.--The Emperor's New Spy (talk) 08:53, 28 September 2012 (UTC)
- No references at all (in Joseph I anyway). Zoonoses (talk) 04:48, 28 September 2012 (UTC)
- Is the Wikipedia article unreferenced on these facts? --Jayron32 03:04, 28 September 2012 (UTC)
- That would be what I want to know.--The Emperor's New Spy (talk) 01:54, 28 September 2012 (UTC)
Peace, man
Why does the dove represent peace in some cultures? All I can think of is it (supposedly) brought back an olive branch to Noah in the Ark - but what does that have to do with it? Rcsprinter (babble) @ 17:13, 27 September 2012 (UTC)
- Have you ever seen a dove killing anyone? --Jayron32 17:18, 27 September 2012 (UTC)
- Our article on doves as symbols ascribes the origin of this to the Noah story. Looie496 (talk) 17:23, 27 September 2012 (UTC)
- (edit conflict) See Peace dove and Doves as symbols. My favourite explanation is from the peace dove article, that the Olive branch is a Pagan symbol of peace, and became associated with the dove through the story of Noah. This makes sense, since when doves are used to denote peace, they are often depicted with an olive branch. Another explanation apparently comes from equation of doves with the Holy Spirit. - Cucumber Mike (talk) 17:26, 27 September 2012 (UTC)
Book about bees
Hi, I recently stumbled upon a (I think) early 20th century book by a French author about bees, but stupidly forgot to bookmark it. It looked interesting though, and was written from a non-technical and possibly slightly romantic perspective. The author just really liked bees. What could it have been? --Iae (talk) 19:32, 27 September 2012 (UTC)
- A quote from a book about bees was posted on an RD in the last few months, talking about how one set of bees (drones?) were, to their great surprise, killed by other bees at the end of the foraging season. Does that ring a bell as the source? --Tagishsimon (talk) 19:38, 27 September 2012 (UTC)
- Could it have been one of the works of Jean-Henri Fabre by any chance? --some jerk on the Internet (talk) 20:13, 27 September 2012 (UTC)
- Some of which can be seen on Google Books. Alansplodge (talk) 20:24, 27 September 2012 (UTC)
- That does ring a bell Tagishsimon, thanks, it's quite possible I got it from here. I'll have a search of the desks. It's not Fabre sadly. I think the author began with 'M', thinking about it. --Iae (talk) 20:31, 27 September 2012 (UTC)
- If it was on these desks, then I know for sure it would have been some time in the weeks leading up to the 25th July (as that's when I went on holiday and lost it as a 'saved tab' in my browser). Searching brings up nothing, but how do I look at the archives for a specific date, rather than by keyword? --Iae (talk) 20:33, 27 September 2012 (UTC)
- Sorry, I worked it out eventually: [16]. The author was Maurice Maeterlinck. Thanks all. --Iae (talk) 20:36, 27 September 2012 (UTC)
- If it was on these desks, then I know for sure it would have been some time in the weeks leading up to the 25th July (as that's when I went on holiday and lost it as a 'saved tab' in my browser). Searching brings up nothing, but how do I look at the archives for a specific date, rather than by keyword? --Iae (talk) 20:33, 27 September 2012 (UTC)
- Could it have been one of the works of Jean-Henri Fabre by any chance? --some jerk on the Internet (talk) 20:13, 27 September 2012 (UTC)
September 28
Math
What is the difference between USAMO and USAJMO beside the obvious fact that USAJMO is for 10 grade or below and USAMO is for anyone who capable of doing it. The USAMO and USAJMO I'm talking are the American Mathematic Contests. I know the top 6 smartest scorer in the USAMO will get to represent the USA in International Mathematical Olympiad. So what do top 6 in USAJMO get? What is the point of doing USAJMO? Just for the sake of practice for USAMO or something?65.128.190.136 (talk) 04:06, 28 September 2012 (UTC)
- For those strange people who need spoon-feeding, USAMO is United States of America Mathematical Olympiad. -- Jack of Oz [Talk] 05:55, 28 September 2012 (UTC)
What's the point of winning the World Junior Chess Championship? It's not like it gives you the right to challenge for the World Chess Championships? Just for the sake of practice for the World Champs or something?
I'll answer your question and mine: see our articles on prestige and (loosely) sport. --Dweller (talk) 11:45, 28 September 2012 (UTC)
Emergency dispatchers
So twice now I have called a local police department and both times the dispatcher has seemed to get off the phone as quickly as possible. The first time I called 911 (UK's 112 999, Canada's 112) to report a funnel cloud and the second time the non-emergency line to report possible drug activity. The first time basically the moment I told the dispatcher about the funnel cloud I got a response of "We are aware of it and have people monitoring the situation" and a prompt end of the call. The second time I reported my suspicions and the dispatcher said they would have someone check it out then promptly said "Alright, I have to go" and hung up. I was under the impression that generally the dispatchers would want to be more thorough in collecting details than that (with the first case asking if I was a spotter, etc, and in the second collecting information about the people involved), so why would they rush to get off the line? Ks0stm (T•C•G•E) 06:50, 28 September 2012 (UTC)
- You mean 911 is the US's 112 not the UK's which is 999. You will have to ask the local authorities or local politicians who set the priorities for the service. Here in UK we get the same complaints and one reason given is the increase in 'emergency' calls being made as a result of people carrying mobile phones. Sussexonian (talk) 07:06, 28 September 2012 (UTC)
- 112 is European, of course, and is an alternative to 999 in the UK. In my only experience of using the emergency number, the operator remained on the line much longer than I expected, in fact, until the police arrived. I think it all depends on the content of your call and an estimate of the danger someone might be in. Dbfirs 07:43, 28 September 2012 (UTC)
- I'm Canadian, and this is the first I've ever heard of "112". The 112 (emergency telephone number) article even claims that 112 redirects to 911, so i would hardly call 112 a Canadian number. We are decidedly a 911 country. Mingmingla (talk) 18:59, 28 September 2012 (UTC)
- 112 is European, of course, and is an alternative to 999 in the UK. In my only experience of using the emergency number, the operator remained on the line much longer than I expected, in fact, until the police arrived. I think it all depends on the content of your call and an estimate of the danger someone might be in. Dbfirs 07:43, 28 September 2012 (UTC)
- It sounds like they were overloaded with calls, so, as soon as they determined that nobody's life was in danger, they went on to the next call. StuRat (talk) 08:10, 28 September 2012 (UTC)
- Sussexonion & StuRat are right IMO. There seem to be quite a lot of negligence calls, usually because the 9 button of the phone is jammed against some object while the phone is in a pocket or backpack (so the owner doesn't even notice). Holding 9 down for 2 seconds will cause a 911 on many mobiles. On the local radio I've heard that these calls are more frequent than real emergency calls. - ¡Ouch! (hurt me / more pain) 08:21, 28 September 2012 (UTC)
- OTOH, it might simply be the case that the funnel cloud may have already been reported to the very same operator, possibly by multiple callers. Same possibly with the drug users. Just a guess though. It might be as well that you actually beat others to the call, and the operator saw his call queue fill up, so he hurried through your report in case there was a more severe emergency among the queued callers, - ¡Ouch! (hurt me / more pain) 08:21, 28 September 2012 (UTC)
- Some refs back up the above re the need to work quickly and the decision-making process: This article gives some interesting statistics on the volume and nature of 911 calls in a small U.S. county (pop. 21062). Dispatchers field 911, numerous other calls. See also What does a 911 dispatcher do. Taknaran (talk) 15:07, 28 September 2012 (UTC)
- Can you clarify why you were calling 911 about "possible drug activity"? Was someone being forced to buy drugs against his will? What made you suspect immanent physical danger? μηδείς (talk) 20:31, 28 September 2012 (UTC)
Property Rights Question
I have a question--if hypothetically, say, someone had an expensive autographed baseball that was his property which he accidentally threw past his neighbor's fence, and the neighbor refuses to give the autographed baseball back, does the owner of the baseball have a right to demand and even take legal action to get his baseball/property back? Has there ever been any similar scenario to this and is there any knowledge or speculation about how courts might rule in such a situation? Futurist110 (talk) 06:53, 28 September 2012 (UTC)
- OK. Doubtless this will turn into a
squabbledebate about legal advice, despite the use of "hypothetically", but a non-advisory answer is possible. And that answer, as usual, is it depends on the jurisdiction. As an example of a similar scenario, under the law of England and Wales, deliberately keeping someone else's property that has accidentally ended up on your land when the undisputed owner has requested its return is theft, and here is a newspaper article showing how the police handle such situations when a complaint is received. For other jurisdictions you would have to check what the applicable law says. As for speculation about how the courts might rule, this would be inappropriate on the RefDesk and not a whole lot of practical use in the real world, either. I'm hoping we can, between us, demonstrate how to handle this question per policy without provoking a removal or a hatting, but I'm not holding my breath. - Karenjc 08:43, 28 September 2012 (UTC)
- Yup... it depends on the Jurisdiction... consider the situation where someone's cow wanders off and ends up in someone else's field. Who owns the cow? The answer has been different in different places (and even in the same place at different times in history). Blueboar (talk) 14:19, 28 September 2012 (UTC)
- The issue of a cow on someone else's field usually arises because one can identify the field, but not the cow. That's not the case if the cattle are branded. This is not some generic ball. I would love to see a reference to a legal case where a uniquely identifiable item was deemed someone else's property because it was inadvertently misplced on someone else's property. μηδείς (talk) 20:28, 28 September 2012 (UTC)
- Yup... it depends on the Jurisdiction... consider the situation where someone's cow wanders off and ends up in someone else's field. Who owns the cow? The answer has been different in different places (and even in the same place at different times in history). Blueboar (talk) 14:19, 28 September 2012 (UTC)
Bart Simpson's grossest foods
Hello L.H. ! . I am trying to add on the § « Références culturelles » of the french version of The Food Wife , & I see that among the grossest foods Bart boasts in his blogs of having tasted are : "leech cheeks, bear oysters, charlie’s trotters, duck butter, krustyburgers" . For "bear oysters", I discard Acanthus mollis, & rather think of testicle (food) , since they are a lot of bears in the Rockies. For "charlie’s trotters", I think of Viet Cong hams (certainly not too fat…) …. For ""duck butter"", I can’t find anything on WP, but I remember in june 2009 I read somewhere that a widely known just-deceased star had the expression « duck butter » ready as an explanation for the young ones he was so fond of when they questionned him. But I can’t find the reference. Can you help me ? . Thanks a lot beforehand. T.Y; Arapaima (talk) 10:42, 28 September 2012 (UTC)
- "Charlie's Trotters" may be a reference to Charlie Trotter, according to a quick search. Would duck butter be foie gras? — Crisco 1492 (talk) 10:49, 28 September 2012 (UTC)
- I think you're barking up the wrong tree. I think they're just joke names. Guessing what the scriptwriters intended (if anything) is always going to be WP:OR unless you have a source. --Dweller (talk) 11:34, 28 September 2012 (UTC)
- They're so surreal you might as well go for the direct equivalents. Joues de sangsue, huitres d'ours, pieds de Charlot, beurre de canard, krustyburgers. Makes absolutely no sense at all, but then it doesn't in English. The hunches about oysters and Charlie Trotter could well be right, but you can't capture everything in translation. Itsmejudith (talk) 13:28, 28 September 2012 (UTC)
- Trotter would usually be pig's feet. Rmhermen (talk) 16:14, 28 September 2012 (UTC)
- Arapaima, have these episodes aired in French yet? You could just use the translated terms. There are transcriptions of most of the French episodes at SimpsonsPark (although not this episode, not yet). Adam Bishop (talk) 16:43, 28 September 2012 (UTC)
Handly's Lessee v. Anthony
Where is the peninsula in question in Handly's Lessee v. Anthony? From the description given in the article, I can't figure it out, because there are many places that fit the general description, and I don't myself know about any meanders that have streams as described here. I've tried Google but found nothing. Anyway, a modern map might not help, because dams built for the Ohio River canalization process have raised the water levels in many places and changed the shorelines accordingly. 2001:18E8:2:1020:60FB:F516:1E7C:B1D3 (talk) 16:04, 28 September 2012 (UTC)
United States electoral college system
As a citizen of the United Kingdom I don't claim to know much about the American electoral system. What I do understand is that states are granted electoral college votes based on their population, and that when a democrat or repubican candidate gets a majority all of the electoral college votes are awarded to that candidate. What I dont understand is in states like California, New York and Florida, where there are a lot of electoral college votes to be obtained, why are the votes not evenly distributed between the two nominees. For instance, lets assume theres 50 million people in California; if 26 million people voted democrat and then 24 million voted republican, why would the democrat candidate get all of the 55 available votes? Surely it should be averaged out between the two candidates. Doesn't make much sense to me but it doesn't seem particularly democratic. --Toryroxy (talk) 17:05, 28 September 2012 (UTC)
- You can read more about it at "Electoral College (United States)", specifically the "Criticism" and "Support" sections. Gabbe (talk) 17:29, 28 September 2012 (UTC)
- With the exception of Maine and Nebraska, you are perfectly correct. It does not make much sense and it is far from democratic. Many Americans realize how antiqued and broken the system is and are working towards fixing it. The National Popular Vote Interstate Compact will effectively abolish the electoral college and it's already 49% to its goal despite being only 5 years old. A8875 (talk) 17:33, 28 September 2012 (UTC)
- Partisan answers aside, the reason that it works that way is that the Constitution permits each state to decide how its electoral votes are cast, and most state legislatures have chosen to use a winner-take-all format. Note that all interstate compacts require the approval of Congress in order to become law. Nyttend (talk) 17:55, 28 September 2012 (UTC)
- I believe the standard argument is that splitting the seats like that would reduce the influence of the state, since most of their seats would just cancel each other out (looking at the 2008 results, apart from DC and a few states with only 3 or 4 seats, the largest majority was 65%, which means only 30% of that state's seats would actually count for anything, and it's far less for most states). The only way to implement a more democratic system without weakening your own position is if everyone else (or, at least, a very large proportion) goes along with it too, which is the point of the interstate pact mentioned above. --Tango (talk) 18:31, 28 September 2012 (UTC)
- The system may not be perfect, but it was a lot more democratic than having a King and a Parliament dominated by the House of Lords determine things... which was how things ran back in England before the Americans declared their Independence. It is also important to remember that, in 1787 (when this was all set up), each State was essentially an independent nation... and they did not fully trust each other. New York, for example, was not about to agree to something that might give more political clout to Virginia. So each State insisted that it set its own rules on how to choose Electors. And it was in each State's interest to have all its Electors form a solid block (all voting for same Presidential candidate), rather than a a divided slate that was split proportionally between more than one candidate. Thus, the preference for "Winner-take-all" in most states. Blueboar (talk) 18:33, 28 September 2012 (UTC)
- Surely now though its time to rethink that as it happened two and a bit centuries ago and now the United States is a more stable political entity. Also the explanation you provided seems to refer mostly to historic states choosing the winner takes all system, I don't see why the western states would adopt this policy. I think it would be better if in the US election you forgot about all the states and the nation voted as a whole. — Preceding unsigned comment added by Toryroxy (talk • contribs) 19:19, 28 September 2012 (UTC)
- Except, you came here to ask why it is the way it is - which we have answered. Not to argue for why it should be changed - which would be a debate, which we cannot accommodate. -- Jack of Oz [Talk] 19:52, 28 September 2012 (UTC)
- I have to agree 100% with Nyttend, Tango, Blueboar, and 141.59% with JackofOz, whose pretty yellow signatures I miss. μηδείς (talk) 20:19, 28 September 2012 (UTC)
- Except, you came here to ask why it is the way it is - which we have answered. Not to argue for why it should be changed - which would be a debate, which we cannot accommodate. -- Jack of Oz [Talk] 19:52, 28 September 2012 (UTC)
Intrinsic vs. Extrinsic religiosity
Can anyone explain to me the difference between intrinsic and extrinsic religiosity? What does it mean 'to be religious to an end'? Psychologically speaking, why is intrinsic religiosity better than extrinsic religiosity or irreligiosity? How does a person become religious? Are some people just born that way? What does that say about apostates (people who leave the faith) and religious converts (people who enter the faith)? My former psychology textbook talked something about intrinsic religiosity vs. extrinsic religiosity. My impression was that being intrinsically religious was better than extrinsically religious or religious to an end. No idea what that meant. If an atheist who embraces secular humanism disguises himself or herself as, say, a Christian and attends a traditional Christian church, then would that count as extrinsic religiosity? What happens if that atheist uses Jesus's teachings to the philosophy of religious humanism? Meanwhile, the Christian who is so-called "born-again" is intrinsically religious, even though he may be explicitly hateful and bigoted and arrogant and selfish and greedy? That does not make sense. 140.254.226.242 (talk) 20:23, 28 September 2012 (UTC)
British royals and languages
Has any British royal ever master or attempt to master all the languages of the British Isles? That is Scottish Gaelic, Welsh, and English (I skipped Lowland Scottish because it is just English with an accent in my own opinion). The Tudors spoke some Welsh but not Gaelic; has any monarch since Elizabeth I of England learn or attempt to learn this language (this part is restricted to actual reigning monarchs sense I now know Prince Charles speaks Welsh)? James IV of Scotland was the last Scottish king to speak Gaelic; has any royal since learn or attempt to learn this language? Also has any British monarch learn or attempt to learn Irish?--The Emperor's New Spy (talk) 20:30, 28 September 2012 (UTC)